Inpatient Medicine




© Springer International Publishing Switzerland 2016
Kevin Conrad (ed.)Absolute Hospital Medicine Review10.1007/978-3-319-23748-0_1


Inpatient Medicine



Laura Bateman  and Kevin Conrad 


(1)
Internal Medicine Department, Ochsner Clinic Foundation, 1521 Jefferson Hwy, New Orleans, 70121, LA, USA

(2)
Department of Hospital Medicine, Ochsner Medical Center, 1521 Jefferson Hwy, New Orleans, 70121, LA, USA

 



 

Laura Bateman



 

Kevin Conrad (Corresponding author)





1.

A 30-year-old female with systemic lupus erythematosus is recovering from a fracture of the right femur and right radius following a motor vehicle accident. She has been in the hospital for 5 days. She has a temperature spike of 39.0 °C (102.2 °F). Blood cultures are drawn which grow yeast, species to be identified. She does not appear ill or toxic. No obvious source of infection is found. The patient has an indwelling central catheter.

In addition to changing the patient’s central line, which of the following do you recommend?

A)

Continued observation.

 

B)

Computed tomography of the chest and abdomen.

 

C)

Start fluconazole.

 

D)

Start liposomal amphotericin B.

 

E)

Repeat blood cultures and treat if positive.

 

 




  • Answer: C


  • Candida is an increasingly common pathogen found due to line infections. It is currently the fourth leading bloodstream pathogen. In this particular case, despite the patient not appearing ill, treatment directed toward Candida should be initiated. The most common antifungal agents used for the treatment of candidemia are fluconazole and the echinocandins. These include caspofungin, micafungin, and anidulafungin. Amphotericin B is given less often due to the risk of nephrotoxicity. Both the echinocandins and the azoles are better tolerated than amphotericin B formulations. Candidemia requires treatment with antifungal agents. Catheter removal alone is not adequate therapy for candidemia. Several studies have noted the high mortality rates associated with candidemia. Furthermore, prompt initiation of therapy is crucial.




  • Reference


  • Manolakaki D, Velmahos G, Kourkoumpetis T, Chang Y, Alam HB, De Moya MM Mylonakis, E. Candida infection and colonization among trauma patients. Virulence. 2010;1(5):367–75.


2.

A 41-year-old male presented to the emergency room with shortness of breath and chest pain. Imaging reveals a pulmonary embolus. He is started on enoxaparin 1 mg/kg SQ BID and warfarin 5 mg PO daily. Nursing staff reports to perform enoxaparin teaching in preparation for his discharge the following day; however, he reports that he is terrified of needles and feels as if twice daily injections will not be possible (weight = 77 kg, CrCl = 89 ml/min).

Are there any other options to decrease the number of injections for this patient?

A)

Enoxaparin 1 mg/kg SQ daily

 

B)

Enoxaparin 0.5 mg/kg SQ daily

 

C)

Enoxaparin 1.5 mg/kg SQ daily

 

D)

Both A and C

 

 




  • Answer: C


  • Enoxaparin 1 mg/kg SQ daily would be used if a patient had a CrCl < 30 ml/min. Enoxaparin 0.5 mg/kg SQ is the indicated dose for infants >2 months and children </=18 years of age for thromboembolism prophylaxis and would be dosed BID, not daily. Enoxaparin 1.5 mg/kg SQ daily is an appropriate outpatient dosing regimen for patients with a CrCl > 30 ml/min for treatment.




  • Reference


  • Garcia DA, et al. CHEST guidelines – parenteral anticoagulants. Chest. 2012;141(2_suppl):e24S–43S.


3.

A 39-year-old female presents to the emergency room with progressive worsening of imbalance and vertigo accompanied by recent falls. She has a 5-year history of multiple sclerosis. Her last admit for a multiple sclerosis flair was 6 months ago. Medications are currently interferon beta-1a and gabapentin for neuropathic pain.

On physical examination, she is afebrile. Blood pressure is 120/66 mmHg. Heart rate is 60 bpm. Internuclear ophthalmoplegia is noted on the left. Gait testing shows imbalance when she walks, which is markedly worse from baseline. An MRI is scheduled for the morning.

Which of the following is the most appropriate first-line treatment?

A)

Oral prednisone 60 mg daily

 

B)

Intravenous methylprednisolone 1 g daily administration

 

C)

Methylprednisolone 125 mg Q 6 h

 

D)

Increase gabapentin dosage

 

E)

Plasmapheresis

 

F)

B and E

 

 




  • Answer: F


  • Intravenous methylprednisolone with a suggested dose of 1 g/day for 3–5 days has been the traditional treatment for acute exacerbations of multiple sclerosis. This patient is experiencing an acute exacerbation or relapse of her underlying multiple sclerosis. The data supports the use of high-dose intravenous corticosteroids. This treatment regimen has been demonstrated to speed the recovery from a multiple sclerosis attacks; however, it is uncertain whether this impacts long-term disability.


  • The 2011 American Academy of Neurology (AAN) Plasmapheresis Guideline Update states that plasmapheresis is effective and may be considered in fulminant demyelinating CNS disease as a first-line agent.


  • Plasmapheresis can also be considered in cases resistant to corticosteroid therapy, and clinical improvement should be followed closely. Previous clinical trials have demonstrated that oral prednisone is inferior to high-dose intravenous corticosteroids. A change in this patient’s chronic disease-modifying therapy may be considered in consultation with a multiple sclerosis specialist, but would be of no benefit in the acute multiple sclerosis flair.




  • Reference


  • Rodriguez M, Karnes WE, Bartleson JD, Pineda AA. Plasmapheresis in acute episodes of fulminant CNS inflammatory demyelination. Neurology. 1993;43(6):1100–4.


4.

A 65-year-old woman presents complaining of severe dizziness. Some mild nausea with eating is reported as well. She notes it especially occurs when she turns over in bed and immediately upon standing. She has had an episode of this before which resolved without medical care. She is currently unable to take care of herself at home and is admitted for further workup and intravenous fluids.

On physical exam, you ask the patient to sit on the bedside with her head turned approximately 45° to the right. You slowly lower the patient to the supine position and extend her head backward 20°. This maneuver immediately reproduces the patient’s dizziness, and you note increased nystagmus.

What is the appropriate management in the evaluation and treatment of this patient?

A)

MRI of her cerebellum

 

B)

Methylprednisolone taper beginning at 60 mg daily

 

C)

Repositioning (Epley) maneuvers

 

D)

Rizatriptan 10 mg orally once

 

E)

Valacyclovir 1000 mg three times daily for 10 days

 

 




  • Answer: C


  • The symptoms and physical examination of this patient are typical of benign paroxysmal positional vertigo (BPPV). Referral to ENT or physical therapy for repositioning maneuvers is the best treatment.


  • BPPV is a common cause of vertigo. Episodes of BPPV are typically brief, lasting no more than 1 min. They are brought about by changes in position. Typical reported movements that elicit vertigo are lying down, rolling over in bed, rising from bed, sitting up and tilting the head to look upward. Vertigo is often accompanied by nystagmus that beats upward and torsionally toward the affected ear. This can be elicited by the Dix–Hallpike maneuver which was performed on physical exam.


  • The history and physical examination are not consistent with a central cause of vertigo. An MRI can probably be avoided. Methylprednisolone is the primary treatment of acute vestibular neuritis. It is of benefit if used within the first 3 days of symptoms. Vestibular neuritis often presents with more prolonged and persistent symptoms. Most patients recover spontaneously, but when used early, methylprednisolone will decrease the duration of symptoms. Antiviral therapy is not indicated unless there is an obvious herpes zoster infection. Likewise, the symptoms are not consistent with migrainous vertigo, which would be persistent for hours and not be affected by positional changes.




  • Reference


  • Epley JM. The canalith repositioning procedure: for treatment of benign paroxysmal positional vertigo. Otolaryngol Head Neck Surg. 1992;107(3):399–404.


5.

A 26-year-old woman presents to the emergency department with shortness of breath as her primary complaint, which has been progressively increasing for several days since starting her menstrual period. She has also been experiencing increasing weakness during the past week as well. She notes a worsening of her symptoms at the end of the day, and she has noticed weakness while brushing her hair. Occasionally she reports blurry vision or difficulty with reading. On physical exam, no specific weakness is noted. She seems fatigued in general and has a depressed affect. Pulmonary exam is normal. All labs are within normal range.

Which of the following neuromuscular disorders is most likely the cause of this patient’s symptoms?

A)

Guillain–Barré syndrome

 

B)

Bilateral diaphragmatic paralysis

 

C)

Myasthenia gravis

 

D)

Duchenne muscular dystrophy

 

E)

Amyotrophic lateral sclerosis (ALS)

 

 




  • Answer: C


  • Multiple neuromuscular disorders may affect respiratory function. Guillain–Barré syndrome usually presents as an ascending paralysis with respiratory symptoms occurring later and rarely as a presenting symptom. Although bilateral diaphragmatic paralysis would explain this patient’s shortness of breath, the proximal muscle weakness and ocular symptoms would remain unexplained. Duchenne muscular dystrophy is an X-linked disorder that exclusively affects males that presents by 12 years of age.


  • The majority of patients with ALS present clinically with progressive asymmetrical weakness, fasciculations, and prominent muscle atrophy. The distal musculature is primarily involved. Myasthenia gravis is an autoimmune disorder that interferes with the postsynaptic acetylcholine receptor. Patients usually present with intermittent symptoms that are usually worse at the end of the day. Respiratory symptoms may be the presenting symptom. The most common severe symptom of myasthenia gravis is respiratory failure. Exposure to bright sunlight, surgery, immunization, emotional stress, menstruation, infection and physical factors might trigger or worsen exacerbations.




  • Reference


  • Keesey JC. Clinical evaluation and management of myasthenia gravis. Muscle Nerve. 2004;29(4):484–505.


6.

A 57-year-old white man is admitted with exertional shortness of breath. His symptom began several months ago and has gotten progressively worse over time. He reports occasional upper respiratory complaints, some fatigue, and a nonpainful, nonpruritic rash on his lower extremities. His medical history is significant only for diabetes.

On physical exam, you note a mildly erythematous, papular rash with a few nodules on his lower extremities. His pulmonary examination is notable for bilateral crackles. Initial workup is unrevealing. A chest radiograph reveals interstitial abnormalities bilaterally. Urine dipstick testing reveals proteinuria and hematuria. A blood test for cytoplasmic antineutrophil cytoplasmic antibodies (c-ANCA) is pending. A subsequent biopsy reveals a necrotizing granulomatous vasculitis.

This patient’s findings are most consistent with which of the following diagnoses?

A)

Lymphomatoid granulomatosis

 

B)

Systemic lupus erythematosus (SLE)

 

C)

Granulomatosis with polyangiitis (GPA)

 

D)

Churg-Strauss syndrome

 

E)

Goodpasture disease

 

 




  • Answer: C


  • This patient has granulomatosis with polyangiitis (GPA), which was formerly known as Wegener’s disease. It is associated with both distinctive and nonspecific mucocutaneous signs. Palpable purpura suggestive of vasculitis is one of the most common skin findings. A variety of other dermatologic conditions have been reported including ulcers, papules, and nodules. In addition to upper and lower pulmonary symptoms, nasal ulcerations and septal perforation should suggest the diagnosis. Biopsy is required. Diagnosis is made by the demonstration of a necrotizing granulomatous vasculitis in a patient with upper and lower respiratory tract disease and glomerulonephritis. C-ANCA autoantibodies make several autoimmune diseases less likely.


  • The absence of asthma makes the diagnosis of Churg-Strauss syndrome unlikely. Patients with lymphomatoid granulomatosis present with a predominance of pulmonary and nervous system manifestations, and tests for ANCA autoantibodies are usually negative.




  • Reference


  • Cartin-Ceba R, Peikert T, Specks U. Pathogenesis of ANCA-associated vasculitis. Curr Rheumatol Rep. 2012;14(6):481–93.


7.

A 68-year-old male was admitted to the hospital for pneumonia. He had a chest X-ray done that reveals a left lung infiltrate and pleural effusion. He underwent a thoracentesis that revealed an exudative effusion. With no resolution of the effusion, a repeat thoracentesis and chest tube was placed 2 days later. Three days later, he is spiking temperatures to 103 °F. Repeat chest X-ray reveals a worsening effusion on the left.

Which of the following is the best option in management at this time?

A)

Expand antibiotic coverage

 

B)

Repeat chest tube placement

 

C)

Change antibiotics and continue with chest tube drainage

 

D)

Perform video-assisted thoracic surgery (VATS)

 

E)

No change required at this time

 

 




  • Answer: D


  • Patients with pneumonia and exudative effusion resistant to drainage should be considered for video-assisted thoracoscopic surgery (VATS).


  • Approximately 15–40% of patients with exudative effusion require surgical drainage of the infected pleural space. Chest tubes often become clogged or effusions become loculated. Patients should be considered for surgery if they have ongoing signs of sepsis in association with a persistent pleural collection. VATS is used as a first-line therapy in many hospitals if cardiothoracic surgery support is available. Open thoracic drainage remains a frequently used alternative technique.




  • Reference


  • Ferguson AD, Prescott RJ, Selkon JB, Watson D, Swinburn CR. The clinical course and management of thoracic empyema. QJM. 1996;89(4):285–9.


8.

You are called to see a patient on the floor for acute obtundation. She has been admitted for chronic abdominal pain and possible pancreatitis. She has been in the hospital for 6 h and has received several doses of narcotics. Her last dose was 4 mg of hydromorphone 15 min ago. She has been on hydromorphone every 4 h. After you assess the patient, you administer a dose of naloxone. The patient has an immediate improvement of her symptoms. She is alert and oriented. One hour later, you are called again to see the patient, who has become somnolent again. The most likely cause of patient’s worsening mental status is:

A)

Worsening CO2 retention

 

B)

Diminishing effects of naloxone

 

C)

Further narcotic use

 

D)

Sepsis

 

E)

Delirium secondary to pancreatitis

 

 




  • Answer: B


  • Naloxone has an extremely rapid onset of action. The duration of naloxone is no greater than 1–2 h. This is of importance as patients who have received a one-time dose may have a return of intoxication symptoms. All opiates have a longer duration of action than naloxone. All patients with opiate overdose symptoms both in the emergency room and on the floor should be monitored for a return of symptoms after naloxone has been given.


  • Naloxone is most commonly injected intravenously for fastest action, which usually causes the drug to act within a minute. When IV access is not available, it can also be administered via intramuscular or subcutaneous injection. In emergency circumstances, it can be administered intranasally.




  • Reference


  • Orman JS, Keating GM. Buprenorphine/naloxone: a review of its use in the treatment of opioid dependence. Drugs. 2009;69(5):577–607.


9.

A 55-year-old female presents with an the inability to close her left eye, mild numbness, and tingling of the left cheek. She has a history of hypertension. You are consulted by the emergency room to admit the patient for probable stroke.

On physical examination, vital signs are normal. No lesions of the skin or mucous membranes are noted. Neurologic examination reveals a weakness of the left upper and lower facial muscles and an inability to close the left eye. Sensory examination reveals the facial sensation is normal bilaterally.

CT scan is performed which reveals no significant abnormalities. Other labs are within normal limits.

Which of the following is the most appropriate treatment?

A)

Acyclovir

 

B)

Intravenous methylprednisolone

 

C)

Aspirin

 

D)

Sumatriptan

 

E)

Prednisone

 

 




  • Answer: E


  • This patient has an acute onset of Bell’s palsy. The current treatment of choice is prednisone. Mounting evidence suggests that Bell’s palsy is due to human herpes virus 1. It is often is seen after a viral prodrome. Physical examination reveals paralysis of both the upper and lower facial motor neurons which distinguishes it from a cerebrovascular accident.


  • The patient may also report dry mouth, impaired taste, and pain and numbness in the ear. Abrupt onset of symptoms usually occurs over 1–2 days. The most appropriate treatment is oral prednisone 40 mg per day, started within the first 72 h. Antiviral agents have been used in the past. There is no evidence to date that anti-herpes virus agents, such as acyclovir, as monotherapy for Bell’s palsy are of benefit. High-dose intravenous corticosteroids are not indicated for the treatment of Bell’s palsy.




  • Reference


  • Baugh RF, Basura GJ, Ishii LE, Schwartz SR, Drumheller CM, Burkholder R, et al. Clinical practice guideline: Bell’s Palsy executive summary. Otolaryngol Head Neck Surg. 2013;149(5):656–63.


10.

A 23-year-old female is brought to the emergency room by her roommate with the chief complaint of increasing agitation, which began this morning. All history is obtained from the roommate who reports that the patient has been out for the past two days and has been acting strange and paranoid since returning home. Nothing is known about what the patient may have ingested.

On physical exam, she is diaphoretic, alert, and agitated. She exhibits some unusual behaviors, such as picking at her legs. Her temperature is 38.3 °C (101.0 °F), heart rate is 120 bpm, and respirations are 26 per minute. Her blood pressure is 165/100 mmHg. On physical exam, it is noted that she has poor dentition. A few pustules and scabs are noted on the face. Cardiopulmonary and abdominal exams are normal. She is slightly hyperreflexic. Laboratory examination reveals normal liver enzymes and a normal basic metabolic profile.

Which of the following substances is the most likely cause of the patient’s clinical picture?

A)

Cocaine

 

B)

Benocyclidine

 

C)

Methamphetamine (Crystal Meth)

 

D)

MDMA (ecstasy)

 

E)

LSD

 

 




  • Answer: C


  • Methamphetamine, which has multiple street names including crystal or crystal meth, presents with hypertension, tachycardia, hyperthermia, and often with poor dentition and evidence of skin excoriations. Other common symptoms are paranoia and diaphoresis. Methamphetamine ingestion is commonly seen in both urban and rural settings. Ingestions often occur at extended late night events, concerts, and within group settings. Methamphetamine users and addicts may lose their teeth abnormally quickly. This may be due to several factors that lead to dry mouth as well as specific behaviors that are induced by the drug.


  • 5–15 % of users may fail to recover completely after cessation of drug use. Antipsychotic medications may effectively resolve the symptoms of acute amphetamine psychosis.




  • Reference


  • Darke S, Kaye S, McKetin R, Duflou J. Major physical and psychological harms of methamphetamine use. Drug Alcohol Rev. 2011;27(3):253–62.


11.

A 28-year-old white male presents to ED with a large swollen left arm and severe left arm pain and redness. He believes the only significant event that happened prior to the swelling was picking up a propane tank with his left arm. Of note, he is a construction worker who operates cranes and is seated for the majority of his workday. He works 6 days a week with Sunday being his only day off. He is eager to get back to work. After initial workup and imaging, a DVT is confirmed in the upper extremity.

What would be an appropriate therapy to start?

A)

Rivaroxaban

 

B)

Warfarin plus Enoxaparin 1 mg/kg SQ BID x minimum 5 days

 

C)

Warfarin alone

 

D)

Enoxaparin 1 mg/kg for 2 weeks alone

 

E)

A or B

 

 




  • Answer: E


  • Rivaroxaban may be a great choice for this patient due to his age and work lifestyle. It can be started without the need for bridging therapy. Coumadin monitoring can be problematic and leads to a great deal of noncompliance.


  • If agreeable to the patient, warfarin plus enoxaparin is also an acceptable option. This dosing regimen is well established and validated.


  • Current guidelines recommend that patients with upper extremity DVTs require treatment similar to lower extremity DVT. Option C is incorrect because of the lack of bridging required initially for this disease state.




  • References


  • Ansel J, et al. Pharmacology and management of the vitamin K antagonists – ACCP evidence-based clinical practice guidelines (8th edition). Chest. 2008;133(6_suppl):160S–98S.


  • The EINSTEIN Investigators. Oral Rivaroxaban for treatment of VTE. N Engl J Med. 2010;363:2499–510.


12.

A 38-year-old female patient with a history of end-stage renal disease on home peritoneal dialysis presents with a chief complaint of new onset abdominal pain. A peritoneal catheter was placed 6 months ago and since that time has had no complications.

On physical exam, the abdomen is diffusely tender. She is afebrile. Serum WBC count is 15,000/μL.

Initial therapy for treatment of suspected peritoneal dialysis-induced peritonitis would include which of the following?

A)

Ceftriaxone

 

B)

Vancomycin plus ceftriaxone

 

C)

Vancomycin, ceftriaxone, and diflucan

 

D)

Vancomycin

 

E)

Ceftriaxone and catheter removal

 

 




  • Answer: B


  • Peritonitis is a common complication seen in peritoneal dialysis patients. Unlike peritonitis seen in end-stage liver disease, the majority of infections are gram-positive bacteria. For this reason, vancomycin or other MRSA-covering antibiotic should be included in the initial therapy. Antifungal therapy should be initiated only if the gram stain reveals yeast. Catheter removal should be considered in certain circumstances but is not necessarily indicated with every infection. Indications for removal of catheter include a repeat infection after 4 weeks of antibiotic therapy, infection not responding to antibiotics, fungal peritonitis, or other resistant causes of peritonitis.




  • Reference


  • Piraino B, Bailie GR, Bernardini J, et al. Peritoneal dialysis-related infections recommendations: 2005 update. Perit Dial Int. 2005;25(2):107–31.


13.

A 65-year-old man with a history of hepatitis C and progressive liver disease presents to the hospital with increasing low-grade fever, abdominal pain, and distension. He is currently on furosemide, spironolactone, and nadolol.

On physical examination, his temperature is 37.5 °C(99.5 °F), blood pressure is 100/50 mmHg. Abdominal examination reveals distended abdomen and marked ascites. The abdomen is mildly tender upon palpation. Creatinine is 0.8 mg/dl and total bilirubin is 2.1 mg/dl.

Abdominal ultrasound is consistent with cirrhosis, splenomegaly, and large volume of ascites. Diagnostic paracentesis is scheduled.

The most appropriate initial treatment is?

A)

Cefotaxime

 

B)

Cefotaxime and albumin

 

C)

Furosemide and spironolactone

 

D)

Large volume paracentesis

 

 




  • Answer: A


  • Spontaneous bacterial peritonitis (SBP) is a common complication of end-stage liver disease. Initial treatment consists of antibiotics that have coverage of gram-negative bacteria. Common isolates are Escherichia coli and Klebsiella pneumonia. There is no evidence that large volume paracentesis improves outcomes in patients with spontaneous bacterial peritonitis. Diagnostic paracentesis should be undertaken to confirm the diagnosis. SBP is confirmed when a WBC count of >250 per microliter is found. Additional paracentesis can be considered to determine the efficacy of treatment or to relieve symptoms.




  • Reference


  • Cholongitas E, Papatheodoridis GV, Lahanas A, Xanthaki A, Kontou-Kastellanou C, Archimandritis AJ. Increasing frequency of Gram-positive bacteria in spontaneous bacterial peritonitis. Liver Int. 2005;25(1):57–61.


14.

A 45-year-old woman is being admitted for continued fever. Her symptoms started five weeks ago with the onset of low-grade daily fever. Over the past 2 weeks, she has developed erythematous rash, fatigue, and weight loss. She has been seen twice by her primary care physician. Limited workup has been unrevealing. Her medical history is only significant for hypertension. She takes lisinopril.

On physical exam, the patient’s temperature is found to be 38.3 °C (101.0 °F), a 2/6 murmur is heard in the mitral area of the chest, and an erythematous rash is noted on both legs. A complete blood count shows anemia. The patient’s erythrocyte sedimentation rate (ESR) is elevated at 80 mm/h. A transthoracic echocardiogram shows a 2 cm pedunculated mass in the left atrium.

Which of the following is the most likely diagnosis?

A)

Metastatic colon adenocarcinoma

 

B)

Cardiac rhabdomyosarcoma

 

C)

Papillary fibroelastoma

 

D)

Cardiac myxoma

 

E)

Endocarditis

 

 




  • Answer: D


  • This patient has an atrial myxoma. Myxomas consist of benign scattered stellate cells embedded in a mucinous matrix. About 70 % of myxomas are in the left atrium. Myxomas often present clinically with mechanical hemodynamic effects, which often simulate mitral or tricuspid stenoses or regurgitation. Systemic symptoms include fatigue, fever, erythematous rash, myalgias, and weight loss, accompanied by anemia and an increased ESR. These symptoms may mimic endocarditis. About 10 % of myxomas are genetic. Surgery is the primary treatment.


  • Cardiac tumors are usually metastatic. Metastatic cardiac involvement occurs 20–40 times more frequently than primary tumors. Eighty percent of all primary cardiac tumors are benign. Myxomas account for more than half of these in adults.




  • Reference


  • Larsson S, Lepore V, Kennergren C. Atrial myxomas: results of 25 years’ experience and review of the literature. Surgery. 1989;105(6):695–8


15.

You are asked to admit a 40-year-old man with atypical chest pain. He reports the abrupt onset of an exertional type of pain. The emergency room staff is concerned that the pain may be angina. On further questioning by you, he reports a constant pain of 4 days duration. The pain is worse with inspiration and is positional. He also reports recent fever.

On physical exam, he has diffuse mild chest wall tenderness but is primarily positional in nature. An ECG shows 2 mm elevation ST elevation in the precordial leads, without reciprocal changes and with PR segment depression in lead 2. An echocardiogram performed in the emergency room is normal. A CT angiogram of the chest is pending.

What is the most likely diagnosis and treatment for this patient?

A)

Acute pericarditis; nonsteroidal anti-inflammatory drug (NSAID) are indicated

 

B)

Acute pericarditis; start prednisone

 

C)

Acute pericarditis; echocardiogram in 1 week to confirm diagnosis

 

D)

Musculoskeletal strain; observation alone

 

E)

Pulmonary embolism

 

 




  • Answer: A


  • The patient has acute pericarditis. The chest pain of acute pericarditis is sudden and severe. It is constant over the anterior chest. In acute pericarditis, the pain worsens with inspiration and is reliably positional. The absence of a significant effusion on echocardiography is not evidence against acute pericarditis. Salicylates or NSAIDs are the first-line agents for treatment. Corticosteroids should be reserved for severe cases that are unresponsive to initial therapy. Symptoms may recur after steroid withdrawal, making their use problematic. Low-grade fever and sinus tachycardia may be present.


  • If carefully auscultated, a pericardial friction rub can be detected in most patients when symptoms are acute. Electrocardiographic changes are common in infectious pericarditis and can occur with other etiologies as well. The characteristic change is an elevation in the ST segment in all leads. The absence of reciprocal ST segment depression distinguishes this characteristic pattern of acute pericarditis from acute myocardial infarction. Depression of the PR interval, which is not as obvious, is often the earliest electrocardiographic manifestation.




  • Reference


  • Maisch B, Seferovic PM, Ristic AD, Erbel R, Rienmüller R, Adler Y, et al. Guidelines on the diagnosis and management of pericardial diseases executive summary; the Task force on the diagnosis and management of pericardial diseases of the European society of cardiology. Eur Heart J. 2004;25(7):587–610.


16.

A 45-year-old-man is brought to the emergency department by his family for lethargy, altered mental status, and abdominal discomfort. His past medical history includes combined diastolic and systolic heart disease with EF of 30 %, diabetes mellitus 2 with a HgA1c of 11 %, and a baseline creatinine of 1.8 units/L checked last week in clinic. However, the patient is not currently taking any medications for diabetic management. His glucose on admission was 450 mg/dL with negative serum ketones. A diagnosis of hyperosmolar nonketotic acidosis was made. The patient was treated with an intensive insulin drip and fluids with resolution of symptoms. A successful transition to basal-prandial insulin regimen is made the next day.

What is the best diabetic regimen to discharge your patient home on?

A)

Metformin BID plus lantus 10U SQ nightly

 

B)

Levemir SQ nightly with prandial novolog with meals

 

C)

Glyburide 5 mg daily and insulin sliding scale

 

D)

Dual oral therapy with metformin and rosiglitazone

 

 




  • Answer: B


  • The patient presented with hyperosmolar nonketotic acidosis and a hemoglobin A1c of 11 % meeting requirements to begin a basal-prandial insulin regimen. In patients with consistent extreme hyperglycemia greater than 300 mg/dl, hgbA1c greater than 10 %, insulin should be started immediately.


  • Metformin is contraindicated in men with a creatinine >1.5 mg/dL and women with creatinine >1.4 mg/dL. The glitazones are typically not recommended for diabetics in the setting of severe uncontrolled hyperglycemia.




  • Reference


  • American Diabetes Association. Standards of Medical Care in Diabetes-2015: Abridged for Primary Care Providers. Clinical Diabetes. 2015. 33(2)


17.

A 27-year-old male is brought to the emergency room after being found down in the parking lot of a grocery store. While in the emergency room, he awakens and reports chest pain. An ECG reveals a 4 mm ST segment elevation in leads II, III, and aVF. Rapid drug screen is positive for cocaine. The emergency room staff administers aspirin, Ativan, and sublingual nitroglycerin. You are consulted for admission. His heart rate is 120 beats per minute. After 5 min, chest pain is not alleviated by nitroglycerin.

Which of the following is the most appropriate treatment?

A)

Enoxaparin

 

B)

Repeat lorazepam

 

C)

IV metoprolol

 

D)

Nitroprusside

 

E)

Urgent coronary arteriography

 

 




  • Answer: E


  • Cocaine often induces vascular syndromes due to increased platelet aggregation and endothelial dysfunction. In this particular case, the patient is suffering an acute myocardial infarction due to cocaine. Urgent coronary arteriography, if available, is recommended. Avoidance of beta-blockers acutely after cocaine use is currently a part of the American Heart Association guidelines. This is due to the possibility that beta-blockers may lead to unopposed alpha-adrenergic stimulation with subsequent worsening coronary vasospasm. There have been no controlled trials on this issue yet. It is important to recognize that cocaine can induce myocardial infarctions and other thrombotic events in a low-incidence population such as the case here.




  • Reference


  • Hiestand BC, Smith SW. Cocaine chest pain: between a (crack) rock and a hard place. Acad Emerg Med. 2011;18(1):68–71.


18.

A 65-year-old man with acute respiratory distress is transferred to the intensive care unit. He has just been intubated and placed on mechanical ventilation for respiratory failure secondary to aspiration pneumonia. Before intubation, his oxygen saturation was 81 % breathing 100 % oxygen with a nonrebreather mask.

On physical examination, temperature is 37.0 °C (98.6 °F), blood pressure is 145/85 mmHg, and pulse rate is 110 bpm. His height is 150 cm (59 in) and his weight is 78.0 kg (154.3 lb). Ideal body weight is calculated to be 50.0 kg (114.6 lb). Central venous pressure is 9 cm H2O. Cardiac examination reveals normal heart sounds, no murmurs, and no rubs. Crackles are auscultated in the right and left lung fields. The patient is sedated. Neurologic examination is nonfocal.

Mechanical ventilation is set on the assist/control mode at a rate of 16/min. Positive end-expiratory pressure is 8 cm H2O, and FIO2 is 1.0.

Which of the following is the correct tidal volume?

A)

300 mL

 

B)

450 mL

 

C)

700 mL

 

D)

840 mL

 

 




  • Answer: A


  • This patient’s presentation is consistent with acute respiratory distress syndrome (ARDS). The most appropriate tidal volume is 300 mL. Survival in ARDS is improved when patients are ventilated with a tidal volume of 6 mL/kg of ideal body weight (IBW). A low tidal volume mechanical ventilation strategy is now the standard of care for ARDS.


  • Lung injury is presumed to arise from repetitive opening and closing of alveoli. Barotrauma may be limited by low tidal volumes. This can be achieved by delivering limited size tidal volumes, minimizing plateau pressure, optimizing PEEP, and reducing FIO2 to less than 0.6. Ideal body weight rather than actual body weight should be used. Use caution in patients who are overweight or edematous. Calculating actual body weight will typically result in inappropriately large tidal volumes.




  • Reference


  • The Acute Respiratory Distress Syndrome Network. Ventilation with lower tidal volumes as compared with traditional tidal volumes for acute lung injury and the acute respiratory distress syndrome. N Engl J Med. 2000;342(18):1301–8.


19.

A 75 year old Vietnamese male presents with a one month history of bilateral hand swelling and recurrent ulcerations. Over that time period he has been treated with several course of oral antibiotics for presumed cellulitis with no improvement. The family reports that he has had several ulcerations on his fingers develop in the past month that have resolved. He is an active gardener and spends several hours per day tending to his plants. He has no other past medical history.

On physical exam one 3x 3cm superative ulceration is noted distally on his right fifth finger. Some erythema around the ulceration is noted. Both hands are markedly swollen. He is admitted and started on IV vancomycin. After three days no improvement is seen. No fever or elevation in WBC is noted.

On day three, one new lesion similar to the previous lesion develops on the dorsum of his hand. On day four another lesion develops proximally on the forearm.

Which of the following is the most likely diagnosis?

A)

Rheumatoid arthritis

 

B)

Cutaneous Sporotrichosis

 

C)

Mycobacterium Marinum

 

D)

Hypersensitivity Reaction

 

E)

Small Vessel Vasculitis

 

 




  • Answer: B


  • A non healing cellulitis with ulceration is suugestive of an atypical bacterial or fungal infection. In this patient with significant environmental exposure and classic lymphatic spread, Sporotrichosis is the likely diagnosis. Sporotrichosis is a subacute infection caused by the saprophytic dimorphic fungus Sporothrix schenckii.

    The characteristic infection involves ulcerative subcutaneous nodules that progress proximally along lymphatic channels. The primary lesion develops at the site of cutaneous inoculation, typically in the distal upper extremities. After several weeks, new lesions appear along the lymphatic tracts. Patients are typically afebrile and not systemically ill. The lesions usually cause minimal pain. Many affected patients have received one or more courses of antibacterial therapy without benefit.

    Sporotrichosis may involve other organs, including the eye, prostate oral mucosa, paranasal sinuses, larynx and joints. In such patients, the clinical manifestations depend on the organs involved.

    Soil, plants, moss and other organic material are common sources. The rose bush thorn has been described as the classic source.

    Sporotrichosis occurs worldwide, with focal areas of hyperendemicity, such as Peru and China.

    Treatment can be lengthy. One recent guideline recommends oral itraconazole 200 mg/d until 2-4 weeks after all lesions have resolved, usually for a total of 3-6 months.




  • Reference


  • Barros MB, de Almeida Paes R, Schubach AO. Sporothrix schenckii and Sporotrichosis. Clin Micro Rev. Oct/2011. 24:633–654.


20.

A 24 year-old presents with rash, hypotension, and fever. One week ago she was involved in a biking accident, where she sustained a laceration to the leg. It did not require sutures. She has had no recent travel, gardening exposure, or exposure to pets. She is up-to-date on all of her vaccinations. She does not use IV drugs.

On examination her heart rate is 120 bpm, blood pressure is 87/58. The leg laceration looks clean with a well-granulated base and no erythema, warmth, or pustular discharge. She does have diffuse erythema that is prominent on her palms, conjunctiva, and oral mucosa. There is some mild desquamation noted on her fingertips.

Laboratory results are notable for a creatinine of 3.0 mg/dL, aspartate aminotransferase of 289 U/L, alanine aminotransferase of 372 U/L, total bilirubin of 2.8 mg/dL, INR of 1.5, and platelets at 82,000/μL. She is started on broad-spectrum antibiotics and IV fluids.

What is the most likely diagnosis?

A)

Sepsis

 

B)

Leptospirosis

 

C)

Staphylococcal toxic shock syndrome

 

D)

Streptococcal toxic shock syndrome

 

E)

Drug reaction

 

 




  • Answer: C


  • She has toxic shock syndrome. The characteristic diffuse rash and systemic symptoms make Staphylococcus the most likely inciting agent. Antibiotic treatment should cover both the leading causes, S. pyogenes and S. aureus. This may include a combination of cephalosporins, vancomycin, or drugs effective against MRSA. The addition of clindamycin may reduce toxin production and mortality. Toxic shock usually has a prominent primary site of infection or source. Staphylococcal toxic shock can be associated with immunosuppression, surgical wounds, or retained tampons. Staphylococcus aureus colonization can incite toxic shock. In certain circumstances and location, it has been suggested that Rocky Mountain spotted fever and leptospirosis which can have a similar presentation be ruled out serologically to confirm the diagnosis. This patient is at very low risk for these diagnoses.




  • References


  • Lappin E, Ferguson AJ. Gram-positive toxic shock syndromes. Lancet Infect Dis. 2009 May. 9(5):281–90.


  • Schlievert PM, Kelly JA. Clindamycin-induced suppression of toxic-shock syndrome-associated exotoxin production. J Infect Dis. 1984;149(3):471.


21.

What is the appropriate rivaroxaban dose for an indication of pulmonary embolism (weight = 77 kg, CrCl = 89 ml/min)?

A)

15 mg PO BID × 3 weeks, then 20 mg PO daily

 

B)

20 mg PO daily

 

C)

15 mg PO daily

 

D)

10 mg PO BID × 3 weeks, then 5 mg PO daily

 

 




  • Answer: A


  • Per package labeling by the pharmaceutical manufacturer, for patients with a CrCl > 30 ml/min and for the treatment of DVT/PE, take 15 mg PO BID × 3 weeks, and then 20 mg PO daily. 15 mg PO daily is a renally adjusted regimen for atrial fibrillation. A 10 mg daily dose is indicated for postoperative VTE prophylaxis.




  • Reference


  • Garcia DA, et al. CHEST guidelines – parenteral anticoagulants. Chest. 2012;141(2_suppl):e24S–43S.


22.

An 18-year-old male is admitted for a 2-day history of fever, abdominal pain, and left knee pain. In the past year, he has had three similar episodes, each lasting 2 days. He feels well between episodes. He takes no medications and reports no other medical history. He is sexually active with one partner.

On physical examination, the temperature is 38.2 °C (100.8 °F), blood pressure is 144/86 mmHg, heart rate is 90/min, and respiration rate is 18/min. There is diffuse abdominal tenderness without rebound. There is no evidence of hepatosplenomegaly. No lymph nodes are palpable. The left knee has a small effusion. Flexion of the knee is limited to 90°. A well-demarcated, raised, erythematous rash is noted on the right lower extremity overlying the shin. It is tender to touch.

Laboratory studies reveal an elevated erythrocyte sedimentation rate of 56 mm/h. Screening antinuclear antibody test results are negative. Urinalysis reveals 1+ protein with no cells or casts.

Which of the following is the most likely diagnosis?

A)

Adult-onset Still disease

 

B)

Crohn’s disease

 

C)

Familial Mediterranean fever

 

D)

Reactive arthritis

 

E)

Gonococcal arthritis

 

 




  • Answer: C


  • Familial Mediterranean fever (FMF) also known as recurrent polyserositis presents with monoarticular arthritis and systemic complaints. It is episodic and recurrent which suggests an autoimmune disease. The symptoms presented here are most compatible with familial Mediterranean fever (FMF). It is an autosomal recessive disorder characterized by recurrent 12–72 h episodes of fever with serositis, synovitis, most often monoarticular. Ten percent of patients experience their first episode in early adulthood. FMF is most prevalent in persons of Mediterranean ethnicity. Laboratory studies are consistent with acute inflammation. Serology results are negative for autoimmune disease. Proteinuria may occur from renal amyloidosis. Colchicine is the standard therapy. It reduces both acute attacks and amyloidosis.


  • Adult-onset Still disease (AOSD) which may have a similar presentation is characterized by fever, rash, joint pain, and serositis. Pleuritis or pericarditis may occur. Fever associated with AOSD is quotidian, lasting less than 4 h, and often peaks in the early evening. The characteristic rash is evanescent, salmon-colored, and not painful. Abdominal pain is rare. Finally, a markedly elevated serum ferritin level occurs in most patients with AOSD.




  • Reference


  • Kuky O, Livneh A, Ben-David A, et al. Familial Mediterranean Fever (FMF) with proteinuria: clinical features, histology, predictors, and prognosis in a cohort of 25 patients. J Rheumatol. 2013;40:2083–7.


23.

Which of the following is NOT consistent with self-induced infection or factitious fever?

A)

Tachycardia with fever

 

B)

Polymicrobic bacteremia

 

C)

Recurrent soft tissue infections

 

D)

Self inoculation with body fluids

 

E)

Healthcare background

 

 




  • Answer: A


  • Factitious fever and self-induced infections are encountered in the hospital with some frequency. The literature suggests an increase in all forms of factitious illness. In factitious fever, high temperatures are often not associated with tachycardia or skin warmth. Many creative methods have been described in the literature to induce an elevated temperature. A high index of suspicion will usually reveal some unusual patterns.


  • Self-induced infection generally occurs by self-injection of body fluids, pyretic substances, or other contaminated materials. This includes various substances including materials contaminated with feces, pure microbiological cultures, coliform bacilli, and foreign proteins. Patients may have serial episodes of unexplained polymicrobial bacteremia or recurrent soft tissue infections. The underlying disorder for factitious fever may be the Munchausen syndrome and the Munchausen syndrome by proxy.




  • Reference


  • Aduan RP, Fauci AS, Dale DC, et al. Factitious fever and self-induced infection: a report of 32 cases and review of the literature. Ann Intern Med. 1979;90(2):230–42.


24.

A 35-year-old male intravenous drug user is admitted for a febrile illness. Endocarditis is suspected but ruled out by blood cultures and echocardiography. He tested negative for HIV 8 months ago. He describes a one week onset of a cold, characterized by subjective fever, fatigue, and aching joints. In the hospital, he develops a morbilliform rash. You are now concerned that the patient may have an acute infection with HIV.

What test or tests should be ordered in diagnosing this patient?

A)

Enzyme-linked immunosorbent assay (ELISA) for HIV antibody

 

B)

CD4+ T cell count

 

C)

Complete blood count for lymphopenia and thrombocytopenia

 

D)

p24 antigen test of HIV RNA

 

E)

HIV ELISA antibody test and a test for p24 antigen of HIV

 

 




  • Answer: E


  • After acquiring HIV, infected persons may develop a nonspecific febrile illness. The incubation period is 7–14 days after acquiring HIV. The symptoms are similar to influenza or mononucleosis in character. Laboratory testing often reveals lymphopenia and thrombocytopenia, but these findings are not diagnostic. Results of HIV ELISA antibody testing are usually negative because it typically takes 22–27 days for the HIV antibody to become positive. The CD4+ T cell count is usually normal at time of seroconversion.


  • The plasma p24 antigen test is highly specific for HIV infection but is not as sensitive as the HIV RNA assay. Patients typically have a high level of viremia. They are highly infectious at this stage. Plasma HIV RNA level of several million HIV RNA copies per milliliter of plasma are usually seen. The combination of a positive HIV RNA test and a negative screening HIV antibody test result confirms the diagnosis of acute HIV infection.




  • Reference


  • Delaney KP, Branson BM, Uniyal A, Phillips S, Candal D, Owen SM, et al. Evaluation of the performance characteristics of 6 rapid HIV antibody tests. Clin Infect Dis. 2011 Jan 15. 52(2):257–63.


25.

A 22-year-old female is admitted with the acute onset of fever, severe throat pain, and inability to handle her oral secretions.

On physical examinations, she has an erythematous oropharynx and cervical lymphadenopathy. The patient has no known history of drug allergy. She is started on an empirical regimen of amoxicillin for streptococcal pharyngitis.

The next day she developed an erythematous maculopapular rash on several areas. The mono spot test comes back positive.

Which of the following statements regarding this patient’s exanthematous drug eruption is true?

A)

Fever is common in viral-related exanthematous eruptions.

 

B)

Systemic corticosteroids are required to treat this drug eruption.

 

C)

In the future she will be able to tolerate all β-lactam antibiotics, including ampicillin.

 

D)

The mechanism of exanthematous eruption caused by ampicillin is mast cell degranulation.

 

E)

This patient’s rash can be expected to be severe.

 

 




  • Answer: C


  • This patient has ampicillin- or amoxicillin-related exanthematous eruption that can frequently occur with mononucleosis. This does not appear to be IgE mediated and is not a penicillin allergy. Patients may receive penicillins in the future. The etiology of the ampicillin rash that occurs in association with a viral infection is unknown.


  • Fever is not associated with simple exanthematous eruptions. These eruptions usually occur within 1 week after the beginning of therapy and generally resolve within 7–14 days. Scaling or desquamation may follow resolution. The treatment of exanthematous eruptions is generally supportive. Oral antihistamines used in conjunction with soothing baths may help relieve pruritus. Topical corticosteroids are indicated when antihistamines do not provide relief. Systemic corticosteroids are used only in severe cases. Discontinuance of ampicillin is recommended.




  • Reference


  • Kagan B. Ampicillin rash. West J Med. 1977;126(4):333–5.


26.

A 44-year-old male is admitted for acute psychosis. He has a history of schizophrenia and has been on various antipsychotics, lithium, and paroxetine. His agitation in the hospital has been difficult to control and has required escalating doses of haloperidol. On the third day of his hospitalization, he develops temperature of 39.6 °C (103.3 °F). Blood pressure is 110/65. Other medications include lithium and valproic acid.

On physical examination, he has generalized tremors, rigidity, agitation, and diaphoresis. These symptoms have increased since admission. Laboratory studies are significant for a creatinine kinase level of 1480 mg/dL.

Which of the following is the most likely diagnosis?

A)

Lithium toxicity

 

B)

Malignant hyperthermia

 

C)

Neuroleptic malignant syndrome

 

D)

Serotonin syndrome

 

E)

Sepsis

 

 




  • Answer: C


  • This patient’s symptoms of fever, tremor, agitation, and rigidity are consistent with the neuroleptic malignant syndrome. This is a potential life-threatening condition. It is characterized by hyperthermia that is accompanied by autonomic dysfunction, as seen in this patient. This syndrome presents as a reaction to new antipsychotic neuroleptic medications or an increase in neuroleptic medications, as is the case here. The most common offending agents are the older antipsychotics such as haloperidol and fluphenazine. Neuroleptic malignant syndrome rapidly develops over a 24-h period and peaks within 72 h.




  • References


  • Gurrera RJ, Caroff SN, Cohen A, et al. An international consensus study of neuroleptic malignant syndrome diagnostic criteria using the Delphi method. J Clin Psychiatry. 2011;72(9):1222–8


  • Trollor JN, Chen X, Sachdev PS. Neuroleptic malignant syndrome associated with atypical antipsychotic drugs. CNS Drugs. 2009;23(6):477–92.


27.

A 27-year-old male is admitted with severe agitation, psychosis, and violent behavior. He was brought to the emergency room 2 h ago by police. Despite 3 mg of lorazepam given 2 h prior, he remains agitated and difficult to control.

On physical examination, he is diaphoretic and is unable to answer questions. He has a heart rate of 170 bpm, blood pressure of 200/110 mmHg, and a marked vertical nystagmus. He is a known user of cannabis, but no other illicit drug history is known.

Which of the following is the most likely drug ingested?

A)

Cocaine

 

B)

Phencyclidine (PCP)

 

C)

Lysergic acid diethylamide (LSD)

 

D)

Heroin

 

E)

Methylenedioxymethamphetamine (MDMA).

 

 




  • Answer: B


  • Phencyclidine, or PCP, often presents with severe agitation, psychosis, and violent behavior. Often, multiple people are required to restrain the patient. Vertical or rotatory nystagmus is a unique finding characteristic of PCP intoxication. Management of phencyclidine intoxication mostly consists of supportive care. Benzodiazepines have been used for agitation and for the treatment of seizures that can occur with PCP ingestion.


  • Cocaine intoxication may present with similar symptoms but is not reported to cause nystagmus. Lysergic acid diethylamide (LSD) is a typical hallucinogen, but no significant amount of violent behavior is reported. MDMA and ecstasy are both hallucinogenic and a stimulant. Violent behavior is not usually seen. Heroin causes a typical opioid symptom profile consisting of constricted pupils, sedation, and respiratory depression.


  • Since its peak use in urban areas, the 1970s PCP use has declined.




  • Reference


  • Zukin SR, Sloboda Z, Javitt DC. Phencyclidine PCP. In: Lowinson JH, Ruiz P, Millman RB, et al., editors. Substance abuse: a comprehensive textbook, 4th ed. Philadelphia: Lippincott Williams & Wilkins; 2005.


28.

A 43-year-old white male presents to the emergency room for intractable nausea and vomiting. During inpatient admission paperwork, you complete his VTE risk assessment and realize he is a high VTE risk. What would you choose for DVT prophylaxis (BMI = 45, CrCl = 75 ml/min)?

A)

Early ambulation

 

B)

Lovenox 40 mg SQ daily

 

C)

Lovenox 40 mg SQ BID

 

D)

Sequential compression device

 

 




  • Answer: C


  • For thromboprophylaxis with fixed-dose enoxaparin, there is a strong negative correlation between total body weight and anti-Xa levels in obese patients. Several prospective trials have examined this issue in patients undergoing bariatric surgery, with inconclusive findings. However, guidelines have stated that increasing the prophylactic dose of enoxaparin in morbidly obese patients (body mass index >/=40 kg/m2) is appropriate. The most common dosing recommendation for this scenario is 40 mg SQ BID. However, other indications, such as bariatric surgery, have recommended 60 mg SQ BID if BMI>/=50 kg/m2. Early ambulation and sequential compression devices alone would not be appropriate DVT prophylaxis for a high VTE risk.




  • References


  • Frederiksen SG, Hedenbro JL, Norgren L. Enoxaparin effect depends on body-weight and current doses may be inadequate in obese patients. Br J Surg. 2003;90:547–8.


  • Garcia DA, et al. CHEST guidelines – parenteral anticoagulants. Chest 2012;141(2_suppl):e24S–43S.


  • Nutescu EA, Spinler SA, Wittkowsky A, Dager WE. Low-molecular-weight heparins in renal impairment and obesity: available evidence and clinical practice recommendations across medical and surgical settings. Ann Pharmacother. 2009;43:1064–83.


29.

A 35-year-old woman with Crohn’s disease presents with a flare of her disease consisting of fever, right lower quadrant pain, guaiac-positive diarrhea, and macrocytic anemia. She states she has lost 20 lbs from her usual weight of 101 lbs. She is still able to tolerate solid food and liquids. Previously, her disease has been limited to the small intestine and terminal ileum.

On physical exam, she has a temperature of 37.9 °C (100.3 °F), active bowel sounds are heard, and she has right lower quadrant tenderness.

Which of the following statements is true for this patient?

A)

The anemia is probably caused by folate deficiency.

 

B)

Sulfasalazine is the first-line therapy.

 

C)

An aminosalicylate (5-ASA) will be required to control this flare.

 

D)

Corticosteroids will be necessary to control her symptoms.

 

E)

She should be hospitalized and given infliximab.

 

 




  • Answer: D


  • This patient has moderate to severe Crohn’s disease. This is based on her symptoms of fever, weight loss, abdominal pain without obstruction, and ability to continue oral intake. For the treatment of moderate to severe Crohn’s disease, the current recommendations include the “top-down” approach. This differs from the conventional step-up approach in that more potent agents are administered initially. For symptoms of this severity, corticosteroids will be necessary. The use of 5-ASA for the treatment of Crohn’s disease is limited. In studies, only a small subset of patients have benefitted from this agent.


  • Infliximab may be used in patients who are not responsive to salicylates, antibiotics, or steroids. Unless the small bowel mucosal disease is very extensive, the macrocytic anemia is most likely caused by a deficiency of vitamin B12, which is absorbed in the terminal ileum.




  • Reference


  • Ford AC, Bernstein CN, Khan KJ, Abreu MT, Marshall JK, Talley NJ, et al. Glucocorticosteroid therapy in inflammatory bowel disease: systematic review and meta-analysis. Am J Gastroenterol. 2011;106(4):590–9.


30.

A 65-year-old female with a history recently diagnosed small cell lung cancer presents with a chief complaint of fatigue, dizziness, and imbalance. Her sodium level on admission is 112 meq/L. She was noted to have a sodium of 142 mmol/L approximately 1 month ago. Her family reports no change in her dietary habits or excessive water intoxication.

On physical exam she appears to be euvolemic.

The most appropriate initial therapy includes:

A)

Slow correction of her sodium with normal saline

 

B)

Free water restriction

 

C)

3 % saline administration with close monitoring

 

D)

Demeclocycline

 

E)

Dexamethasone

 

 




  • Answer: C


  • In a setting of significant hyponatremia with neurologic symptoms, consideration must be given to administering hypertonic saline. This patient’s condition is almost certainly due to syndrome of inappropriate antidiuretic hormone (SIADH) due to her small cell lung cancer. Isotonic normal saline would probably worsen the hyponatremia by the mechanism of water retention and sodium excretion. When hypertonic saline is administered, careful monitoring of her sodium levels should be done to prevent central pontine myelinolysis, which can occur with rapid correction. In less severe cases with no neurologic findings, water restriction would be the first line of treatment.




  • Reference


  • Zenenberg RD, Carluccio AL, Merlin MA. Hyponatremia: evaluation and management. Hosp Pract. 2010;38(1):89–96.


31.

A 65-year-old male with a recent hospitalization for total knee replacement presents with the chief complaint of eight bowel movements per day. He reports a fever as high as 37.8 °C.

On physical exam, mild abdominal distention is noted, and he appears in no apparent distress. WBC is 16, 000 cells/μl.

Initial therapy for suspected Clostridium difficile disease would include the following:

A)

Oral metronidazole 500 mg q 8

 

B)

Oral vancomycin 125 mg q 6

 

C)

IV metronidazole 500 mg q 6

 

D)

Oral vancomycin 125 g q 6

 

 




  • Answer: D


  • The treatment for Clostridium difficile infections is changing as new protocols and therapies are developed. The patient in this question is characterized as having moderate Clostridium difficile infection. First-line treatment is based on the severity of illness and initial clinical response. Symptoms of moderate disease include 6–12 bowel movements per day, fever of 37.5–38.5 °C, a WBC count between 15,000 and 25,000 cells/μl, or visible GI bleeding. Oral vancomycin is the treatment of choice for moderate disease. For severe disease, oral vancomycin 125 mg q 6 and IV metronidazole 500 mg q 6 is the treatment of choice. For mild disease, oral Flagyl can be used alone, 500 mg q 8. New treatment modalities, such as fidaxomicin as well as stool replacement therapy, have shown promising results and are currently undergoing trials.




  • Reference


  • Ananthakrishnan AN. Clostridium difficile infection: epidemiology, risk factors and management. Nat Rev Gastroenterol Hepatol. 2011;8(1):17–26.


32.

A 54-year-old man is admitted with abdominal pain. He had a similar episode 6 months ago for which he was seen in the emergency room several days after the onset of pain and was discharged home without a definitive diagnosis. He has a history of poorly controlled diabetes mellitus. He has pain for the past 3 days. He denies any alcohol use, which is confirmed by family members.

On physical exam, he is tachycardic and has diminished bowel sounds and epigastric tenderness. He has a papular rash on his knees.

Initial laboratory studies are significant for the following: leukocytes, 16,000 cells/mm3; blood glucose level of 400 mg/dl. An amylase level is normal.

Which of the following is the most likely diagnosis for this patient?

A)

Acute or chronic idiopathic pancreatitis

 

B)

Gallstone pancreatitis

 

C)

Alcoholic pancreatitis

 

D)

Pancreatitis secondary to hypertriglyceridemia

 

E)

Malignancy-induced pancreatitis

 

 




  • Answer: D


  • This patient has triglyceride induced pancreatitis. The serum amylase level may be normal in some patients with acute pancreatitis associated with high triglycerides as marked elevations in the triglyceride level can interfere with the laboratory assay for amylase. The presence of a papular rash on this patient is consistent with eruptive xanthomas due to hypertriglyceridemia.


  • In the acute phase, the initial treatment of hypertriglyceridemia-induced acute pancreatitis focuses on good hydration and analgesia and is similar to the management of acute pancreatitis due to any etiology. The triglyceride levels usually rapidly decrease within 48 h of the onset of acute pancreatitis.


  • Gallstones and alcohol abuse combined account for 70–80 % of all cases of acute pancreatitis. Other etiologies include sphincter of Oddi dysfunction, strictures of the pancreatic duct, congenital anatomic abnormalities and genetic disorders, drugs, toxins, trauma, infections, and metabolic causes. Some cases are idiopathic. Metabolic causes of acute pancreatitis include not only hypertriglyceridemia but hypercalcemias well. Serum triglycerides generally need to be in excess of 1,000 mg/dl to produce acute pancreatitis. This is most commonly seen in type V hyperlipoproteinemia and is usually associated with diabetes mellitus. Acute pancreatitis can itself raise triglyceride levels, but not to this degree.




  • References


  • Suang W, Navaneethan U, Ruiz L, et al. Hypertriglyceridemic pancreatitis: presentation and management. Am J Gastroenterol. 2009;104:984–91.


  • Toskes PP. Hyperlipidemic pancreatitis. Gastroenterol Clin North Am. 1990;19:783–91.


33.

An 80-year-old man has been admitted with a urinary tract infection. On the third day from his admission, he develops the acute onset of chest pain of 30 min duration. He has a history of an inferior myocardial infarction 3 years ago. His medical history is remarkable for hypertension and an ischemic stroke 8 years ago. Current medications include atenolol and aspirin.

On physical exam, the patient is afebrile, his blood pressure is 170/100 mmHg, his pulse is 90 beats/min, and his respiratory rate is 20 breaths/min. He is diaphoretic and in apparent pain. ECG reveals 0.2 mm elevations in leads V2–V6.

Which of the following features, in this case, would be an absolute contraindication to thrombolytic therapy?

A)

Failure to meet ECG criteria.

 

B)

Age greater than 75 years.

 

C)

History of stroke.

 

D)

Elevated blood pressure.

 

E)

There are no absolute contraindications.

 

 




  • Answer: E


  • In this case, there are no absolute contraindications to thrombolytic therapy.


  • The patient meets ECG criteria for the administration of thrombolytic therapy. This includes ST segment elevation greater than 0.1 mm in two contiguous leads.


  • Age greater than 75 years is not a contraindication to thrombolysis. In patients older than 75 years, there is an increased risk of hemorrhagic stroke. Overall mortality is reduced in such patients without other contraindications. A prior history of hemorrhagic stroke is an absolute contraindication to thrombolytic. A history of an ischemic stroke less than 1 year is an absolute contraindication. A stroke more than 1 year before presentation is a relative contraindication. Blood pressure >180/110 mmHg is a relative contraindication to thrombolytic therapy.




  • Reference


  • O’Gara PT, Kushner FG, Ascheim DD et al. 2013 ACCF/AHA guideline for the management of ST-elevation myocardial infarction: a report of the American College of Cardiology Foundation/American Heart Association Task Force on Practice Guidelines. Circulation. 2013;127:e362–452.


34.

A 65-year-old male presents with progressive shortness of breath over the past month. He has a 40 pack-year history of smoking. CT scan of the chest reveals a right middle lobe mass for which he subsequently undergoes biopsy, which reveals adenocarcinoma. Magnetic resonance imaging of the brain reveals a 1 cm tumor in the left cerebral cortex, which is consistent with metastatic disease. The patient has no history of seizures or syncope. The patient is referred to outpatient therapy in the hematology/oncology service as well as follow-up with radiation oncology. The patient is ready for discharge.

Which of the following would be the most appropriate therapy for primary seizure prevention?

A)

Seizure prophylaxis is not indicated.

 

B)

Valproate.

 

C)

Phenytoin.

 

D)

Phenobarbital.

 

E)

Oral prednisone 40 mg daily.

 

 




  • Answer: A


  • There is no indication for antiepileptic therapy for primary prevention in patients who have brain metastasis who have not undergone resection. Past studies have revealed no difference in seizure rates between placebo and antiepileptic therapy in patients who have brain tumors. Antiepileptic therapy has high rates of adverse reactions and caution should be used in their use.




  • Reference


  • Sirven JI, Wingerchuk DM, Drazkowski JF, Lyons MK, Zimmerman RS. Seizure prophylaxis in patients with brain tumors: a meta-analysis. Mayo Clin Proc. 2004;79(12):1489–94.


35.

A 52-year-old male presents with new onset hemoptysis and shortness of breath. He is recovering from an ankle fracture. CT angiography reveals an intraluminal defect in the left lower lobar pulmonary artery and right upper lobe subsegment.

What additional measurements from CT angiography have been shown to have prognostic significance?

A)

Clot burden (e.g., amount of clot seen)

 

B)

Clot location

 

C)

Right versus left ventricular volume

 

D)

Clot size

 

E)

Collateral flow

 

 




  • Answer: C


  • Risk stratification for pulmonary embolism is the goal of many current studies. A reliable method is currently being developed. In patients who are hemodynamically stable, a number of tests have been examined in this effort. Clot size and location are not good at predicting mortality or right ventricular strain.


  • Examination of ventricular volumes comparing right to left may be the best measurement for predicting outcomes. A ratio >1.2 that is suggestive of right ventricular strain has utility in predicting adverse outcome and death. Echocardiograms may look for right heart strain as well. Other tools that are being considered include biomarkers such as troponin and pro-brain natriuretic peptide (proBNP) levels, and clinical models such as PESI (Pulmonary Embolism Severity Index) and PREP (prognostic factors for PE).




  • References


  • Aujesky D, et al. Derivation and validation of a prognostic model for pulmonary embolism. Am J Respir Crit Care Med. 2005;172:1041–6.


  • Becattini C, et al. Acute pulmonary embolism: external validation of an integrated risk stratification model. Chest. 2013. doi:10.​1378/​chest.​12-2938.


  • Sanchez O, et al. Prognostic factors for pulmonary embolism: the PREP study, a prospective multicenter cohort study. Am J Respir Crit Care Med. 2010;181:168–73.


36.

A 68-year-old male with a history COPD presents with a 3-day history of worsening shortness of breath and fever. On the day prior to presentation, he developed abdominal pain and diarrhea. He is employed as an air conditioner repair technician.

On physical exam, his temperature is 39.5 °C (103.1 °F), pulse is 72, and respiratory rate is 30. He is in moderate respiratory distress. Oxygen saturation is 95 % on 2 l of oxygen. He has moderate abdominal pain.

Laboratory data is significant for mild elevation of his transaminases and a sodium of 128. Chest radiograph reveals bilateral infiltrates.

The most appropriate antibiotics and treatment are:

A)

Vancomycin 1 g q12 and piperacillin/sulbactam 4.gms q 8

 

B)

Ceftriaxone 1 g q 12 and azithromycin 500 mg q8

 

C)

Vancomycin 1 g q12/gentamicin

 

D)

Ceftriaxone 1 g q12/Prednisone 40 mg QD

 

E)

Bactrim

 

 




  • Answer: B


  • This patient has legionella pneumonia, which should be treated with a quinolone or macrolide antibiotic. Legionella pneumonia presents with the common symptoms of fever, chills, and cough. Distinguishing features are loss of appetite, loss of coordination, and occasionally diarrhea and vomiting. Relative bradycardia has traditionally been considered a symptom. Laboratory tests may show significant changes in renal functions, liver functions, and electrolytes. This can include marked hyponatremia. Chest X-rays often show bi-basal consolidation. It is difficult to distinguish Legionnaires’ disease from other types of pneumonia by symptoms alone. Serology is often required for diagnosis. Many hospitals utilize the urinary antigen test for initial detection when legionella pneumonia is suspected.


  • It is not spread from person to person, but rather often through exposure to aerosolized cool water, such is the case here.


  • It acquired its name after a July 1976 outbreak of a then unrecognized disease, which afflicted 221 persons, resulting in 34 deaths. The outbreak was first noticed among people attending a convention the American Legion.




  • References


  • Fraser DW, Tsai T, Orenstein W, et al. Legionnaires’ disease: description of an epidemic of pneumonia. N Engl J Med. 1977;297:1186–96.


  • Woo AH, Goetz A, Yu VL. Transmission of Legionella by respiratory equipment and aerosol generating devices. Chest. 1992;102(5):1586–90.


37.

You are called to the floor to see a 77-year-old man who has recently passed a large amount of red and maroon blood per rectum. He was admitted 2 days ago for a urinary tract infection. After this episode, the patient feels dizzy but is conscious and able to converse.

On physical exam, his blood pressure is 100/60 mmHg and the pulse rate is 110/min. He has no abdominal pain, nausea, vomiting, fever, or weight loss. He had a colonoscopy 1 year ago that showed a benign polyp and extensive diverticulosis. He has a single 22 g IV access.

Stat laboratory studies reveal a hemoglobin of 7.5 g/dL. The leukocyte count is 5600/μL. Prothrombin time and activated partial thromboplastin times are normal.

Which of the following is the most appropriate next step in the management of this patient?

A)

Colonoscopy

 

B)

Esophagogastroduodenoscopy

 

C)

Increased intravenous access

 

D)

Placement of a nasogastric tube with lavage

 

E)

Technetium-labeled red blood cell scan

 

 




  • Answer: C


  • This patient is volume depleted and not hemodynamically stable. Survival may be dependent upon the correct management sequence. Two large-bore peripheral catheters or a central line for volume repletion is urgently required. Fluid resuscitation should be started as well as an urgent type and match for blood transfusion. Although rapid diagnosis may be of benefit, early resuscitation measures should not be delayed by diagnostic workup.


  • A nasogastric tube may be considered after volume resuscitation, if an upper source of gastrointestinal bleeding is likely. Regardless of the source of bleeding at this point in the management, the first rule is to achieve hemodynamic stability.


  • Although a colonoscopy is the diagnostic test of choice to evaluate for sources of lower gastrointestinal bleeding, this should not occur before volume resuscitation. A bleeding scan may be indicated if an endoscopic evaluation is not immediately possible or if an endoscopic evaluation has been non-revealing. Diverticular bleed is certainly a possibility in this case. If a bleeding diverticulum is detected on colonoscopy, it can be treated with thermal coagulation or epinephrine injection. Up to 90 % of diverticular bleeding resolves without intervention.




  • References


  • Laine L, Shah A. Randomized trial of urgent vs. elective colonoscopy in patients hospitalized with lower GI bleeding. Am J Gastroenterol. 2010;105(12):2636–41.


  • Scottish Intercollegiate Guidelines Network (SIGN). Management of acute upper and lower gastrointestinal bleeding. A national clinical guideline. SIGN publication; no. 105. Edinburgh (Scotland): Scottish Intercollegiate Guidelines Network (SIGN); 2008.


38.

An 81-year-old female who is admitted to the hospital with a diagnosis of healthcare-associated pneumonia (HCAP). She is empirically started on vancomycin, ciprofloxacin, and piperacillin/tazobactam.

In treating this patient, what is the vancomycin trough goal for HCAP?

A)

10–15 ug/ml

 

B)

15–20 ug/ml

 

C)

25–30 ug/ml

 

D)

28–32 ug/ml

 

 




  • Answer: B


  • Healthcare-associated pneumonia (HCAP) treatment with vancomycin requires higher trough levels of 15–20 mcg/ml.




  • References


  • ATS Board of Directors and IDSA Guideline Committee. Guidelines of the management of adults with hospital-acquired, ventilator-associated, and healthcare-associated pneumonia. Am J Respir Crit Care Med. 2005;171:388–416.


  • Ryback M et al. Therapeutic monitoring of vancomycin in adult patients. Am J Health-Syst Pharm. 2009;66:82–98.


39.

A 40-year-old male is admitted to the hospital for new-onset fever and chills. He denies any other symptoms. He was recently diagnosed with non-Hodgkin’s lymphoma, for which he received his first cycle of chemotherapy 10 days ago. He currently does not have an indwelling venous catheter.

On physical examination, the temperature is 39.0 °C (102.2 °F). Blood pressure is 120/75 mmHg. There is no evidence of mucositis. Chest exam is normal. Heart examination is normal as well. Abdominal exam reveals normal bowel sounds and is nontender. Laboratory data shows a hemoglobin of 10.8 g/dL and leukocyte count of 600/mcL. The differential is 10 % neutrophils and 90 % lymphocytes. Chest X-ray is normal. Blood and urine cultures are pending.

Which is the most appropriate treatment course?

A)

Begin vancomycin.

 

B)

Await blood cultures and urine cultures.

 

C)

Begin vancomycin, amphotericin, and acyclovir.

 

D)

Begin piperacillin/tazobactam.

 

E)

Begin vancomycin, amphotericin, acyclovir, and diflucan.

 

 




  • Answer: D


  • This patient is neutropenic and febrile and warrants rapid initiation of antibiotics. A stepwise and logical approach is needed for the selection of antibiotics. Febrile neutropenia is a medical emergency. Patients may not present with overt signs of infection. However, septic shock and death can occur within hours of presentation.


  • Initial antibiotic choices may include a third-generation cephalosporin, penicillin with beta-lactamase inhibitor, or cefepime. Endogenous flora from the gastrointestinal tract is the probable cause for most cases of febrile neutropenia. Vancomycin may be considered but alone would not be sufficient coverage for this patient. Empiric antifungal therapy is usually reserved for patients who are febrile after the source of infection is not found following 4–7 days of broad-spectrum antimicrobial therapy. Viral infections are not common, and empiric therapy with antiviral agents such as acyclovir is not warranted.




  • Reference


  • Hughes WT, Armstrong D, Bodey GP, et al. 2002 guidelines for the use of antimicrobial agents in neutropenic patients with cancer. Clin Infect Dis. 2002;34(6):730–51.


40.

An 82-year-old female presents to the emergency department complaining nausea and vomiting. She is admitted for dehydration due to probable viral gastroenteritis. On admission her serum creatinine is noted to be 3.0 mg/dL, her baseline is 1.2 mg/dl.

After transfer to the floor, she acutely develops dizziness with a heart rate of 34/min. 12-lead ECG shows marked sinus bradycardia without any ST segment changes. Stat glucose is 95 mg/dl.

Home medications include atenolol 100 mg daily and hydrochlorothiazide 25 mg daily. Blood pressure is 85/68 mmHg.

Which of the following medications is the most appropriate to administer next?

A)

Intravenous 50 % dextrose solution and insulin

 

B)

Intravenous glucagon

 

C)

Intravenous calcium gluconate

 

D)

Intravenous magnesium sulfate

 

E)

Intravenous atropine

 

F)

B or E

 

 




  • Answer: F


  • This patient has bradycardia and hypotension from beta-blocker toxicity. This is caused by a reduced clearance of atenolol, which is renally excreted and impaired because of her prerenal kidney failure. Glucagon is used to reverse beta-blocker toxicity and is often used as a first-line agent when beta-blocker toxicity is the confirmed issue. Atropine may be used according protocols as well.


  • She should also receive fluid resuscitation with intravenous normal saline solution. If she does not improve with glucagon and IV fluids, external pacemaker or transvenous pacemaker can also be used.




  • Reference


  • Hoot NR, Benitez JG, Palm KH. Hemodynamically unstable: accidental atenolol toxicity?. J Emerg Med. 2013;45(3):355–7.


41.

A homeless man is found unconscious by police. He is admitted for hypothermia and possible cellulitis of his left foot. His past medical history is unknown.

On examination, the foot appears atypical for cellulitis. It has hemorrhagic vesicles distributed throughout the foot distal to the ankle. The left foot is cool and has no sensation to pain or temperature. The right foot is hyperemic but does not have vesicles and has normal sensation. The remainders of the physical examination findings are normal. He is started on antibiotics, and further therapy is considered.

Which of the following statements regarding the management of his foot is true?

A)

Rewarming should not be attempted.

 

B)

Heparin has been shown to improve outcomes.

 

C)

Surgical consultation and debridement are indicated.

 

D)

Normal sensation is likely to return with rewarming.

 

E)

Antibiotics improves limb survival.

 

F)

During the period of rewarming, intense pain will occur.

 

 




  • Answer: F


  • This patient presents with frostbite of the left foot. One of the common presenting symptoms of this is sensory changes that affect both pain and temperature reception. Hemorrhagic vesicles are caused by injury to the vasculature. The prognosis is more favorable when the presenting area has a rapid return to normal temperature and color returns as well. Treatment of extremities is with rapid rewarming, which usually is accomplished with a 37–40 °C (98.6–104 °F) water bath.


  • The period of rewarming can be intensely painful for the patient, and often narcotic analgesia is warranted and should be anticipated to improve compliance. If the pain is intolerable, the temperature of the water bath may be lowered slightly. Compartment syndrome can develop. Rewarming should be closely followed. No medications have been shown to improve outcomes. This includes heparin, steroids, calcium channel blockers, and hyperbaric oxygen. Emergent surgical decisions about the need for debridement should be deferred until the boundaries of the tissue injury are determined. Neuronal injury often occurs with abnormal sympathetic tone in the extremity. This may be permanent or resolve over the course of several months.




  • References


  • McCauley RL, Hing DN, Robson MC, Heggers JP. Frostbite injuries: a rational approach based on the pathophysiology. J Trauma. 1983;23(2):143–7.


  • Twomey JA, Peltier GL, Zera RT. An open-label study to evaluate the safety and efficacy of tissue plasminogen activator in treatment of severe frostbite. J Trauma. 2005;59(6):1350–4; discussion 1354–5.


42.

An 18-year-old female is admitted for observation after sustaining a head-to-head blow during a high school soccer game. She briefly lost consciousness on the field but was able to walk on the sidelines without assistance. She was immediately brought to the emergency room and subsequently admitted to the hospital medicine service.

After an overnight stay in the observation unit, she appears back to her usual baseline mental status. Physical examination in the morning is within normal range. Neurologic examination is within normal limits as well.

Which of the following is the most appropriate next step in management?

A)

CT of the head and return to competition if normal

 

B)

Observation for 24 more hours

 

C)

Exclusion from competition for 1 week

 

D)

Clearance for return to competition

 

E)

Funduscopic examination

 

 




  • Answer: C


  • This patient has a grade 3 concussion. A concussion is defined as trauma-induced alteration in mental status that may be associated with transient loss of consciousness. Neither a grade 1 nor a grade 2 concussion involves a loss of consciousness. A grade I concussion, or mild bruising of brain tissue, is the most common form of head injury. The athlete may briefly appear or act confused; however, he or she is able to remember all events following the impact. The difference between a grade II and a grade I concussion is the presence of post-traumatic amnesia. A grade 3 concussion, such as is seen in this patient, is defined by a brief or prolonged loss of consciousness. Current recommendations state that a grade 1 concussion are permitted to return to the contest on the same day as the injury. The athlete should be removed from competition for at least 20 min and examined every 5 min.


  • Those with grade 2 or grade 3 concussions are prohibited from returning that day. Grade 3 concussions are prohibited from returning to competition until the athlete is asymptomatic for 1 week. Hospitalization is indicated in the presence of traumatic findings, abnormal neuroimaging studies, or with persistent abnormalities seen on physical examination.




  • Reference


  • Giza CC, Kutcher JS, Ashwal S, Barth J, Getchius TS, Gioia GA, et al. Summary of evidence-based guideline update: evaluation and management of concussion in sports: report of the Guideline Development Subcommittee of the American Academy of Neurology. Neurology. 2013;80(24):2250–7.


43.

A 77-year-old male presents to the emergency room with a chief complaint of syncope while getting out of bed to go to the bathroom at 2:00 AM. He has no recollection of the event. He was found by his wife on the floor confused. He has no prior history of syncope and denies chest pain.

On physical examination, the patient is alert and oriented to person, place, time, and event. Temperature is 36.6 °C (97.9 °F). Pulse rate is 60 per minute. Respirations are 16 per minute. Blood pressure is 110/40. Cardiopulmonary and neurologic exams are normal. A small laceration to the chin is noted, but otherwise, there is no evidence of trauma to the head.

Which of the following diagnostic test has the highest yield for determining this patient’s cause of syncope?

A)

Measurement of postural blood pressure

 

B)

Cardiac enzymes

 

C)

Ultrasonography of the carotid arteries

 

D)

Computed tomography

 

E)

Electroencephalography

 

 




  • Answer: A


  • Syncope is a common admission to the hospital. The majority of cases are vasovagal in origin and warrant limited and focused workup. History and physical examination are the most specific and sensitive ways of evaluating syncope. These measures, along with 12-lead electrocardiography (ECG), are the only current level A recommendations listed in the 2007 American College of Emergency Physicians (ACEP) Clinical Policy on Syncope.


  • Several tests commonly ordered have little yield. Cardiac enzymes determine the etiology of syncope in only 0.5 % of patients. Carotid ultrasonography determines the etiology of syncope in 0.8 % of patients. Computed tomography of the brain determines the etiology of syncope in 0.5 % of patients. Electroencephalography determines the etiology of syncope in only 0.8 % of patients. The use of these tests is warranted only when there is evidence to suggest that vasovagal syncope is not the cause.


  • A common scenario is for vasovagal syncope to occur in the middle of the night while going to the bathroom, as is the case here. Confusion is often present and does not necessarily point toward a postictal state. Reassurance and lifestyle modifications are the best treatment. This includes methods to reduce nighttime bathroom use, which is common cause of syncope and injury inducing falls.


  • Measurement of postural blood pressure may confirm the diagnosis and is often a cost-effective test in determining the cause of syncope.




  • Reference


  • Huff JS, Decker WW, Quinn JV, et al. Clinical policy: critical issues in the evaluation and management of adult patients presenting to the emergency department with syncope. Ann Emerg Med. 2007;49(4):431–44.


44.

A 50-year-old male is admitted with complaints of moderate mid-epigastric pain in his upper abdomen for a few weeks. He reports moderate heartburn for the past 2 months. He also complains of weight loss of 10 lbs in the last 2 months as well. He does not take any medications except occasional ibuprofen for back pain.

On physical exam he has moderate tenderness in the epigastric area. The patient’s amylase and lipase are normal. CT scan of the abdomen is normal.

What is the next appropriate step in the management of this patient?

A)

Start PPI.

 

B)

H. pylori treatment.

 

C)

Stop ibuprofen.

 

D)

Manometry studies.

 

E)

Upper endoscopy.

 

 




  • Answer: E


  • Many patients with gastroesophageal reflux disease are appropriately treated with empiric therapy. Endoscopy is reserved for those with chronic symptoms who are at risk for Barrett’s esophagus and those with alarm symptoms.


  • Alarm symptoms include dysphagia, odynophagia, gastrointestinal bleeding or anemia, weight loss, and chest pain. Other alarm features are age greater than 55 years and family history of gastric cancer. This patient’s weight loss would warrant endoscopic evaluation.




  • Reference


  • DeVault KR, Castell DO. Updated guidelines for the diagnosis and treatment of gastroesophageal reflux disease. Am J Gastroenterol. 1999;94:1434–42.


45.

An EEG showing triphasic waves is most suggestive of which of the following clinical disorders?

A)

Brain abscess

 

B)

Herpes simplex encephalitis

 

C)

Locked-in syndrome

 

D)

Metabolic encephalopathy

 

E)

Nonconvulsive status epilepticus

 

 




  • Answer: D


  • Triphasic waves have been associated with a wide range of toxic, metabolic, and structural abnormalities. They were first described in a patient with hepatic encephalopathy.


  • The EEG can often provide clinically useful information in comatose patients. Certain EEG patterns may help in determining diagnosis and prognosis. The EEG becomes slower as consciousness is depressed, regardless of the underlying cause. The EEG is usually normal in patients with locked-in syndrome and helps in distinguishing this disorder from the comatose state. Epileptiform activity characterized by bursts of abnormal discharges containing spikes or sharp waves may be useful to diagnose and treat unrecognized nonconvulsive status in a presumed comatose patient. Patients with herpes simplex encephalitis may show a characteristic pattern of focal, often in the temporal regions or lateralized periodic slow-wave complexes.




  • Yang SS, Wu CH, Chiang TR, et al. Somatosensory evoked potentials in subclinical portosystemic encephalopathy: a comparison with psychometric tests. Hepatology. 1998;27:357–9.


46.

Blood cultures should be obtained in which patients admitted for cellulitis?

A)

Presence of lymphedema

 

B)

Liver cirrhosis

 

C)

Presence of ipsilateral orthopedic implant

 

D)

Leukocytosis of <13.5 × 106 μL

 

E)

A, B, and C

 

F)

All of the above

 

 




  • Answer: E


  • Blood cultures are not beneficial for many patients admitted with uncomplicated cellulitis. They should be limited to certain high-risk populations. Any form of immunosuppression or underlying structural damage would increase the risk of bacteremia, and thus a blood culture should be performed and may be of benefit.




  • Reference


  • Phoenix G, Das S, Joshi M. Diagnosis and management of cellulitis. BMJ (Clinical Research ed.). 2012;345:e4955.


47.

A 31-year-old woman presented to the emergency room with a history of low-grade intermittent fever and reported joint pain, swelling, and rapid onset of decreased mental status over the past 2 days. Per family, the joint pains have developed gradually over the past 4 months. In the past 36 h, she first developed profound personality changes that included agitation and mild visual hallucinations. Increasing lethargy followed this.

On physical exam she appears obtunded. Joint tenderness is difficult to assess due to decreased mental status. Mild swelling is noted in several joints.

A CT scan of the brain reveals possible diffuse mild cerebral edema.

Magnetic resonance imaging reveals diffuse microinfarcts. Hemoglobin, WBC, and platelet counts are within normal range. The CSF report 110 lymphocytes and an elevated protein.

The most likely diagnoses is?

A)

Herpes encephalitis

 

B)

Lupus cerebritis

 

C)

Endocarditis

 

D)

Lyme disease

 

E)

Drug injection

 

 




  • Answer: B


  • Lupus cerebritis can pose as a major diagnostic challenge, as many lupus patients have underlying neuropsychiatric symptoms. The case here has some classical findings, but many cases are elusive. Patients may present with acute confusion, lethargy, coma, chronic dementia, depression, mania, affective disturbances, or psychosis.


  • Prompt identification can be extremely difficult, mainly because there is no single laboratory or radiological confirmatory test. Inflammatory markers can be variable. Lupus cerebritis should be included as a possible diagnosis in any young female patient who presents with complicated neurologic manifestations and no alternative diagnosis.




  • Reference


  • Calabrese LV, Stern TA. Neuropsychiatric manifestations of systemic lupus erythematosus. Psychosomatics. 1995;36:344–8.


  • Greenberg BM. The neurologic manifestations of systemic lupus erythematosus. Neurologist. 2009 May. 15(3):115–21


48.

Which of the following regimens are most appropriate for the treatment of Clostridium difficile infections?

A)

Moderate to severe initial episode: vancomycin 125 mg QID for a total of 10–14 days

 

B)

Severe initial episode complicated with shock and megacolon: vancomycin 125 mg po QID plus metronidazole 500 mg Q8 h IV.

 

C)

Severe initial episode but with a complete ileus: consider rectal instillation of vancomycin

 

D)

None of the above

 

E)

All of the above

 

 




  • Answer: E


  • The following are 2010 guidelines for the treatment of the first episode of Clostridium difficile colitis:


  • First episode with mild or moderate leukocytosis with a white blood cell count of 15,000 cells/mL or lower and a serum creatinine level less than 1.5 times the premorbid level – metronidazole 500 mg 3 times per day by mouth for 10–14 days


  • First episode, severe and leukocytosis with a white blood cell count of 15,000 cells/mL or higher or a serum creatinine level greater than or equal to 1.5 times the premorbid level – vancomycin 125 mg 4 times per day by mouth for 10–14 days


  • First episode, severe and complicated by hypotension or shock, ileus, megacolon – vancomycin 500 mg 4 times per day by mouth or by nasogastric tube, plus metronidazole 500 mg every 8 h intravenously


  • If complete ileus, consider adding rectal instillation of vancomycin.




  • Reference


  • Cohen SH, Gerding DN, Johnson S, Kelly CP, Loo VG, McDonald LC, Pepin J, Wilcox MH. Clinical Practice Guidelines for Clostridium difficile Infection in Adults: 2010 Update by the Society for Healthcare Epidemiology of America (SHEA) and the Infectious Diseases Society of America (IDSA). Infect Control Hosp Epidemiol. 2010;3:431–55.


49.

A 52-year-old man is evaluated in the emergency department for a 2-week history of fatigue and nonspecific arthralgia. He reports some increasing shortness of breath over the past few days and now has some pleuritic chest pain. He has a history of coronary artery disease and hypertension. His medications include diltiazem, hydralazine, aspirin, and isosorbide dinitrate.

On physical examination, his temperature is 37.2 °C (99 °F), blood pressure is 145/90 mmHg, pulse rate is 80 bpm, and respiration rate is 24/min. Cardiac examination is normal. Pulmonary examination reveals a mild left pleural friction rub. There are small bilateral knee effusions. A nonblanching purpuric rash is noted over the distal upper and lower extremities.

Laboratory studies show hemoglobin of 7.9 g/dL, leukocyte count 2,200/μL, platelet count 124,000/μL, and erythrocyte sedimentation rate 88 mm/h. Urinalysis reveals 1+ protein, 2–5 erythrocytes/hpf, and 5–10 leukocytes/hpf.

Chest radiograph reveals small bilateral effusions.

Which of the following is the most appropriate diagnostic test to perform next?

A)

Serum and urine electrophoresis

 

B)

Bone marrow aspiration and biopsy

 

C)

CT of the chest, abdomen, and pelvis

 

D)

Rheumatoid factor and anti-cyclic citrullinated peptide antibody

 

E)

Antinuclear antibody and anti-double-strand DNA antibody assay

 

 




  • Answer: E


  • This patient has drug-induced lupus erythematosus (DILE). The most common drugs that cause DILE are hydralazine, procainamide, quinidine, isoniazid, diltiazem, and minocycline.


  • He has new-onset fever, arthralgia, myalgia, nonblanching purpuric rash, pleuritis, pancytopenia, and proteinuria with active urine sediment.


  • Testing for antinuclear antibodies (ANA), as well as anti-double-stranded DNA antibodies and complement levels, is indicated. This multiorgan pattern is suggestive of an autoimmune disorder, in particular of systemic lupus erythematosus (SLE). No specific criteria establish the diagnosis of DILE. Excluding other underlying autoimmune diseases must first be done. SLE is typically ruled out first.


  • Drugs that cause DILE may take months to years before the associated symptoms occur. In addition similar drugs can also induce flairs of SLE.




  • References


  • Fritzler MJ. Drugs recently associated with lupus syndromes. Lupus. 1994;3(6):455–9.


  • Lowe G, Henderson CL, Grau RH, Hansen CB, Sontheimer RD. A systematic review of drug-induced subacute cutaneous lupus erythematosus. Br J Dermatol. 2011;164(3):465–72.


50.

A 28-year-old woman was admitted due to severe head trauma after a motor vehicle accident. Three weeks after admission, there has been no change in her mental status. All vital signs are normal as well as laboratory values.

She is noted to have spontaneous eye opening and is able to track an object visually at times. She does not speak or follow any commands. She in intubated but is fed through a gastrostomy tube. She moves extremities spontaneously but without purposeful movement.

What term best describes this patient’s condition?

A)

Coma

 

B)

Locked-in

 

C)

Minimally conscious state

 

D)

Persistent vegetative state

 

E)

Vegetative state

 

 




  • Answer: E


  • A vegetative state “of wakefulness without awareness” was first described in 1972. In the vegetative state, patients may open their eyelids occasionally and demonstrate sleep-wake cycles, but completely lack cognitive function, communication, or purposeful movement. In addition, extensive neurologic and medical test must be made to rule out treatable causes.


  • In the minimally conscious state, unlike the vegetative state, there is evidence that patients are aware of themselves and/or their environment.


  • Traditionally, per informal US guidelines, a vegetative state that lasts greater than 1 month is considered to be a persistent vegetative state. A diagnosis of persistent vegetative state does not absolutely imply permanent disability because in very rare cases patients can improve, reaching a minimally conscious state or a higher level of consciousness.




  • Reference


  • Ashwal S. The Multi-Society Task Force On Pvs. Medical aspects of the persistent vegetative state – second of two parts 1994. N Engl J Med. 330(22):1572–9.


51.

An 87-year-old female was admitted to the hospital for a heart failure exacerbation. At baseline, she could ambulate, but needed help with some activities of daily living. She has ischemic cardiomyopathy, coronary artery disease, hypertension, and hyperlipidemia. Current medications are furosemide, lisinopril, metoprolol, aspirin, atorvastatin, and heparin given subcutaneously twice daily for deep venous thrombosis prophylaxis.

Since admission, the patient has expressed her concern about receiving heparin injections. She has had a moderate amount of bruising on her abdomen, which is painful. She has asked her nurse several times if she really needs “those shots,” and the nurse has relayed her concerns to you.

On physical examination, heart rate is 82 beats per minute and blood pressure is 120/65 mmHg. Crackles are still heard a third the way up in both lung fields. Ecchymosis are seen on the abdomen. There is edema (1+) extending to the knees. She can walk slowly but safely with a walker or assistance.What is the best treatment?

A)

Continue the heparin, and explain its necessity to the patient.

 

B)

Stop the heparin, and start enoxaparin daily.

 

C)

Stop the heparin, and start venous foot pumps.

 

D)

Start Coumadin.

 

E)

Stop the heparin and encourage the patient to walk with her family.

 

 




  • Answer: E


  • The current guidelines recommend that adults older than age 40 who are hospitalized for medical reasons and are expected to be less mobile for 3 days or more be given some form of deep venous thrombosis (DVT) prophylaxis. This is based on several randomized controlled trials.


  • However, whether these recommendations should apply to older adults is less uncertain. A systematic review and meta-analysis examined the evidence for harm and efficacy of pharmacologic prophylaxis of DVT in older adults. For the most part older adults with comorbidities have been excluded from studies. The majority of events prevented in studies are asymptomatic DVTs. There is no consistent reduction in fatal pulmonary embolism or mortality. When data from the three trials with patients older than age 75 are pooled, a similar reduction in endpoints to other trials is seen. However, two-thirds of these events are asymptomatic DVTs. In the general population, the absolute bleeding risk is generally increased by 2 % in the heparin treatment group. Older age with its comorbidities may increase this risk.


  • The current data suggest that this patient would gain a very small absolute risk reduction for symptomatic DVTs and an even smaller risk for pulmonary embolism. Given her wishes, it is reasonable to stop heparin and encourage ambulation.




  • References


  • Greig MF, Rochow SB, Crilly MA, Mangoni AA. Routine pharmacological venous thromboembolism prophylaxis in frail older hospitalized patients: where is the evidence? Age Ageing. 2013;42:428–34.


  • Wakefield TW, Proctor MC. Current status of pulmonary embolism and venous thrombosis prophylaxis. Semin Vasc Surg. 2000;13(3):171–81.


52.

A 26-year-old female presents with cellulitis of her left forearm. The patient has a history of IV heroin abuse but denies any recent heroin use. Current medications are lorazepam and methadone.

On presentation, her temperature is 35.8 °C (96.4 °C), respirations are 10 per minute, and blood pressure is 124/72 mmHg. Electrocardiogram reveals wide complex variable focus tachycardia. Toxicology screen is positive for cannabis, alcohol, and opiates.

Which of the following medications is the most likely cause of this patient’s arrhythmia?

A)

Cannabis

 

B)

Benzodiazepines

 

C)

Alcohol

 

D)

Methadone

 

E)

Oxycodone

 

 




  • Answer: D


  • Cardiac arrhythmias in methadone users have been reported for several decades. The most serious have been wide complex tachycardias. Risk factors include female sex, hypokalemia, high-dose methadone, drug interactions, underlying cardiac conditions, and unrecognized congenital long Q-T interval syndrome.


  • In methadone patients, an ECG should be obtained on admission. QT prolongation may predict subsequent malignant arrhythmias and possible need to alter methadone treatment. Methadone has been used to treat heroin addicts for nearly 50 years, but little is known about its long-term side effects.




  • Reference


  • Justo D, Gal-Oz A, Paran Y et al. Methadone-associated torsades de pointes (polymorphic ventricular tachycardia) in opioid-dependent patients. Addiction. 2006;101:1333–8.


53.

A 38-year-old man is admitted with atypical chest pain of 3 h duration. While waiting to be seen, his pain resolves. He reports that he has been smoking marijuana extensively and denies any other ingestions or substances.

He reports no past medical history and is not on any meds. He is not sure but may have chest pain prior to this admission.

On physical exam, his heart rate is 110, temperature is 37 °C (98.6 °F), and blood pressure is 180/90. He is 96 % on room air. He is agitated, tachycardic, and diaphoretic. His electrocardiogram reveals slight ST depressions in leads V3 through V5. Initial troponin level is 0.1 mg/mL. Toxicology screen is pending.

Initial therapy should include the following:

A)

Tissue plasminogen activator (TPA)

 

B)

Percutaneous transluminal coronary angioplasty (PTCA)

 

C)

Aspirin

 

D)

Abciximab

 

E)

Metoprolol

 

 




  • Answer: C


  • This patient is exhibiting a sympathomimetic presentation probably due to crack cocaine ingestion. This should be considered as an additive in someone with a history of smoking marijuana. His possible myocardial ischemia is due to endothelial dysfunction as well as aggregation of platelets. In this particular scenario, there is no evidence of segment elevation myocardial infarction, so PTCA, tPA, and GP IIb/IIIa are not indicated. A reasonable approach would be an aspirin with further cardiovascular workup considered.


  • Beta-blockers are relatively contraindicated in cocaine-induced chest pain as there is a possible risk of increased peripheral vascular resistance. This has not been well tested.




  • References


  • Hobbs WE, Moore EE, Penkala RA, Bolgiano DD, López JA. Cocaine and specific cocaine metabolites induce von Willebrand factor release from endothelial cells in a tissue-specific manner. Arterioscler Thromb Vasc Bio. 2013;33:1230–7.


  • 2014 AHA/ACC Guideline for the Management of Patients With Non-ST-Elevation Acute Coronary Syndromes: A Report of the American College of Cardiology/American Heart Association Task Force on Practice Guidelines. Circulation. 2014;130:e344–e426.


54.

A 75-year-old male with Parkinson’s disease is admitted for worsening tremor and confusion. On his first night of hospitalization, he is noted to be markedly agitated. Which of the following drugs should not be used for treatment?

A)

Haloperidol

 

B)

Olanzapine

 

C)

Risperidone

 

D)

All of the above

 

 




  • Answer: D


  • Delirium and psychosis occur in about one-third of patients with Parkinson’s disease. There have been no conclusive studies on the best approach in the treatment of delirium in patients with Parkinson’s disease.


  • There is a strong contraindication for haloperidol, olanzapine, and risperidone because of potential exacerbation of extrapyramidal symptoms. There is also a lack of efficacy reported with olanzapine. Other atypical antipsychotics such as quetiapine have been commonly used with caution and slow titration. Most studies suggest that quetiapine is safe in Parkinson’s disease. Some reports suggest that there is an increased risk of adverse motor effects in these patients, predominantly in demented subjects.




  • Reference


  • David A. Quetiapine in the treatment of psychosis in Parkinson’s disease. Ther Adv Neurol Disord. 2010;3(6):339–50.


55.

Which of the following options are appropriate prophylaxis of venous thromboembolism for hospitalized medical patients with a moderate risk of bleeding undergoing dialysis three times per week?

A)

Aspirin

 

B)

Warfarin to maintain INR between 1.5 and 2.5

 

C)

Heparin 5000U SQ TID

 

D)

Lovenox 40 mg SQ daily

 

E)

Lovenox 30 mg SQ daily

 

 




  • Answer: C


  • Venous thromboembolism, which includes pulmonary embolism and deep venous thrombosis, continues to be a common clinical problem. The American College of Physicians recommends pharmacologic prophylaxis with heparin or a related drug for venous thromboembolism in medical patients unless the assessed risk for bleeding outweighs the likely benefits. Aspirin and mechanical prophylaxis with graduated compression stockings have been shown to provide inferior coverage in comparison but in some circumstances may be the best option. Heparin 5000U SQ TID is recommended for patients with end-stage renal disease undergoing dialysis. Lovenox is cleared by hemodialysis in an irregular manner and is contraindicated in patients undergoing dialysis.




  • Reference


  • Qaseem A, et al. Venous thromboembolism prophylaxis in hospitalized patients: a clinical practice guideline from the American College of Physicians. Ann InternMed. 2011;155(9):625–32.


56.

A 66-year-old man with history of diabetes, hypertension, and coronary artery disease is admitted with sepsis. Blood cultures drawn on day 1 started growing enterococci on day 3. He was also diagnosed with an infective endocarditis and was started on gentamicin and aqueous penicillin G.

His labs at the time of admission showed a BUN of 32 mg/dL and serum creatinine of 0.8 mg/dL. He started complaining of worsening shortness of breath on the next day, and a CT angiogram of the chest was obtained and pulmonary embolism was ruled out. His BUN on day 7 was 53 mg/dL with a serum creatinine of 2.8 mg/dL. Urine output dropped to 600 cc in 24 h. His vital signs and urinalysis were normal.

What is the reason for this acute renal failure?

A)

Aminoglycoside toxicity

 

B)

Drug-induced interstitial nephritis

 

C)

Acute glomerulonephritis

 

D)

Diabetic nephropathy

 

E)

Contrast-induced nephropathy

 

 




  • Answer: E


  • This patient has developed acute renal failure secondary to contrast-induced nephropathy. Several causes of renal failure are possible. They include interstitial nephritis secondary to penicillin and infective endocarditis. Aminoglycoside toxicity should also be considered.


  • Normal complement levels (C2,3,4) and rapid onset point toward contrast-induced nephropathy. Aminoglycoside toxicity usually happens 4–5 days after the therapy.




  • Reference


  • Murphy SW, Barrett BJ, Parfrey PS. Contrast nephropathy. J Am Soc Nephrol. 2000;11(1):177–82.


57.

A 75-year-old female was admitted to the hospital 3 days prior for community-acquired pneumonia. She has a history of hypertension, hyperlipidemia, and peripheral vascular disease. Medications on admission are lisinopril, metoprolol, hydrochlorothiazide, pravastatin, and aspirin. On admission, Zosyn and vancomycin were initiated. She underwent a CT angiogram to rule out pulmonary embolism. She is now afebrile. Her blood pressure is 110/55 mmHg. She has no evidence of orthostasis. Since admission, her respiratory status has improved and her creatinine level has increased from a baseline of 1.5–3.2 mg/dL. Laboratory studies reveal a urine sodium of 44 mEq. Her fractional excretion of sodium is 2 %, and her fractional excretion of urea is 50 %.

Which of the following is the most likely cause of this patient’s acute renal injury?

A)

Cholesterol emboli

 

B)

Acute interstitial nephritis

 

C)

Prerenal azotemia

 

D)

Normotensive ischemic acute kidney injury

 

E)

Contrast dye-induced kidney injury

 

 




  • Answer: D


  • This patient has normotensive ischemic kidney injury. The findings of elevated fractional excretion of sodium, fractional excretion of urea, and granular casts seen on urinalysis are all consistent with this diagnosis.


  • The patient’s medical history reveals evidence of an underlying chronic kidney disease possibly due to vascular disease. This places the patient at increased risk for normotensive ischemic injury. The patient’s lower blood pressure during hospitalization may be the result of a variety of factors, including infection, better medicine compliance, and the low-salt diet typically seen during a hospitalization.


  • Acute interstitial nephritis, which is often caused by a hypersensitive reaction to medication, is a possibility. However, the patient’s lack of fever, rash, and leukocytes on her urinalysis argues against this diagnosis. Prerenal azotemia is common in the consideration of this patient’s differential diagnosis. The patient’s fractional excretion of sodium of 2 % and fractional excretion of urea of 50 % argues against this. The fractional excretion of urea is a more sensitive test for patients on diuretics.




  • Reference


  • Abuelo JG. Normotensive ischemic acute kidney injury. N Engl J Med. 2007;357(8):797–805.


58.

A 72-year-old female was admitted with an ankle fracture. One hour after receiving a dose of morphine, she developed the acute onset of diffuse abdominal pain. She has a history of known cardiovascular disease and hepatitis C. Her current medications are atenolol, aspirin, and lisinopril.

On physical examination, her temperature is 36.7 °C(98.0 °F), blood pressure is 84/60 mmHg. Abdominal examination reveals diffuse abdominal tenderness upon palpation. No guarding or rebound is noted. No ascites is noted.

CT scan reveals small bowel wall thickening and intestinal pneumatosis. Her WBC count is 14,000 μm/l, and an elevated serum lactate is noted. The most likely diagnosis is:

A)

Pancreatitis

 

B)

Crohn’s disease

 

C)

Acute mesenteric ischemia

 

D)

Spontaneous bacterial peritonitis

 

E)

Infectious ileitis

 

 




  • Answer: C


  • Acute mesenteric ischemia (AMI) is a syndrome caused by inadequate blood flow through the mesenteric vessels from a combination of preexisting vascular disease, emboli, and hypotension. This results in ischemia and eventual gangrene of the bowel wall. It is a potentially life-threatening condition. This patient’s drop in blood pressure due to morphine triggered the ischemic event.


  • AMI may be classified as either arterial or venous. CT scan and laboratory values in this case are consistent with an acute event. CT scan may reveal bowel wall thickening or, in some instances, intestinal pneumatosis as in this case. Treatment options for acute thrombosis can be surgical, stenting, or thrombolytics. Early and aggressive diagnostic imaging and early surgical consultation are warranted. Angiography is the test of choice for both diagnosis and possible therapeutic vasodilation and stenting.


  • Because of the high mortality and the difficulty of diagnosis, mesenteric ischemia has traditionally been considered a diagnostic challenge.




  • Reference


  • Boley SJ, Brandt LJ, Sammartano RJ. History of mesenteric ischemia. The evolution of a diagnosis and management. Surg Clin North Am. 1997 Apr. 77(2):275–88.


59.

A 91-year-old African American female was admitted from her nursing home for altered mental status and foul-smelling urine.

UA is positive with 3+ leukocytes and many bacteria. The admitting physician empirically started moxifloxacin 400 mg IV daily. You are assuming care the following day. Her vital signs are stable, and she seems in no distress.

What would be the next best step?

A)

Continue current regimen.

 

B)

Discontinue moxifloxacin and start ceftriaxone.

 

C)

Continue moxifloxacin but change to PO.

 

D)

None of the above.

 

 




  • Answer: B


  • Moxifloxacin is a quinolone antibiotic that does not achieve adequate concentration in the urine, thereby eliminating its use in the treatment of urinary tract infections. It does not matter whether moxifloxacin is given either oral or IV – the bioavailability of the oral is 100 % and the IV formulation has no ability to concentrate in urine.




  • Reference


  • Gupta K, Hooton TM, Naber KG et al. International clinical practice guidelines for the treatment of acute uncomplicated cystitis pyelonephritis in women: a 2010 Update by the Infectious Disease Society of America and the European Society of Microbiology and Infectious Disease. Clin Inf Dis. 2011;52:e103–20.


60.

You are called in consultation to see a 35-year-old male who was in a motor vehicle accident and underwent surgical repair of a right femur fracture. Postoperatively, the patient has received acetaminophen and scheduled doses of oral morphine. He has become acutely agitated and is oriented only to person. The only admission labs were a complete blood count and a basic metabolic panel, which are normal. His past medical history is not known.

On physical examination, his temperature is 39.0 °C (102.1°), pulse rate is 110 beats per minute, and respirations are 18 per minute. Blood pressure is 180/90 mmHg. The lungs are clear upon auscultation. There are no signs of infection noted. The patient seems agitated with a mild tremor. He is diaphoretic.

Which of the following is the most likely diagnosis?

A)

Drug-induced delirium from morphine

 

B)

Fat emboli

 

C)

Alcohol withdrawal

 

D)

Pneumonia

 

E)

Deep wound infection

 

 




  • Answer: C


  • Alcohol use disorders are common and can complicate postoperative recovery. In this particular case, the fever, tachycardia, hypertension, and tremor are suggestive of alcohol withdrawal. The alcohol level in trauma patients should be checked on admission, as withdrawal is a common source of delirium in this population.


  • About 9 % of US adults meet the criteria for an alcohol use disorder. Less than 50 % of alcohol-dependent persons develop any significant withdrawal symptoms that require pharmacologic treatment upon cessation of alcohol intake. Minor withdrawal occurs within 6–24 h following the patient’s last drink and is characterized by tremor, anxiety, nausea, vomiting, and insomnia. Major withdrawal occurs 10–72 h after the last drink. The signs and symptoms include visual and auditory hallucinations, whole body tremor, vomiting, diaphoresis, and hypertension.


  • The most objective and best-validated tool to assess the severity of alcohol withdrawal is the Clinical Institute Withdrawal Assessment for Alcohol.




  • Reference


  • Mayo-Smith MF, Beecher LH, Fischer TL, et al. Management of alcohol withdrawal delirium. An evidence-based practice guideline. Arch Intern Med. 2004 Jul 12. 164(13):1405–12.


61.

A 57-year-old man is admitted for recent onset of fatigue and weakness. He has been seen twice in the past 2 months as an outpatient for similar symptoms with no diagnoses made. The family states that he is currently unable to take care of himself. The patient further reports nocturia, polyuria, and weight loss over the past 3 months. He has COPD and a 58-pack-year smoking history.

On physical examination, the temperature is 36.4 °C (97.5 °F), blood pressure is 178/97 mmHg, pulse rate is 86/min, and respiration rate is 24/min. Proximal muscle weakness is noted in the upper and lower extremities. Hyperpigmented mucous membranes are noted.

Laboratory studies are as follows: creatinine is 1.4 mg/dL, sodium is 149 mEq/L, glucose is 273, urine cortisol is 472ug per 24 h, and ACTH is 257 pg/ml.

Chest radiographs show hyperinflated lung fields but no masses.

Which of the following is the most likely cause of this patient’s findings?

A)

Adrenal adenoma

 

B)

Adrenal carcinoma

 

C)

Ectopic ACTH secretion

 

D)

Pituitary adenoma

 

E)

New onset diabetes

 

 




  • Answer: C


  • This patient has Cushing syndrome due to excessive ACTH production. This is almost certainly due to underlying malignancy such as lung cancer which is the case here. Approximately half of all cases of ectopic ACTH secretion are due to small cell lung cancer, which has a long list of paraneoplastic syndromes associated with it.


  • Adrenal adenomas can be associated with hypercortisolism, but the features tend to cause a mild Cushing syndrome. Adrenal adenomas are associated with suppressed ACTH levels. Hyperpigmentation suggests excessive ACTH production.


  • A chest radiograph does not rule out the possibility of a lung malignancy and computed topography of the chest is needed.




  • Reference


  • Iias I, Torpy DJ, Pacak K, Mullen N, Wesley RA, Nieman LK. Cushing’s syndrome due to ectopic corticotropin secretion: twenty years’ experience at the National Institutes of Health. J Clin Endocrinol Metab. 2005;90(8):4955–62.


62.

A 67-year-old female is admitted to the hospital service with an unexpected syncopal episode. There are no factors to suggest a vasovagal episode. She reports worsening shortness of breath with exertion over the past 3 months. Otherwise, she has enjoyed good health.

Physical exam is pertinent for a 3/6 systolic crescendo-decrescendo murmur at the left sternal border with radiation to the carotids. You suspect aortic stenosis as the cause of the syncope and order a 2D cardiac echo with color flow Doppler. Results of the 2D echo indicate aortic stenosis.

Which of the following are indications to refer your patient for evaluation for aortic valve replacement?

A)

Exertional dyspnea

 

B)

Aortic valve mean pressure gradient of 40 mmHg or higher

 

C)

Aortic stenosis in the setting of LVEF less than 50 %

 

D)

All of the above

 

E)

A and B

 

 




  • Answer: D


  • Aortic valve replacement is recommended for symptomatic patients with severe aortic stenosis. Severe aortic stenosis is defined as an aortic velocity 4.0 m per second or greater or by a mean pressure gradient 40 mmHg or higher. Symptoms of heart failure, syncope, exertional dyspnea, angina, or presyncope by history or on exercise testing are also indications.




  • Reference


  • Nishimura R, et al. 2014 AHA/ACC guidelines for the Management of patients with Valvular Heart Disease. A Report of the American College of Cardiology/American Heart Association Task Force on Practice Guidelines. J Am Coll Cardiol. 2014;63:57–185.


63.

A 71-year-old woman is admitted from a nursing home with confusion, fever, and flank pain. She has a presumed urinary tract infection.

On physical exam, temperature is 38.8 °C (101.8 °F), blood pressure is 86/50 mmHg, pulse rate is 125/min, and respiration rate is 24/min. Mucous membranes are dry, and there is costovertebral angle tenderness, poor skin turgor, and no edema.

Hemoglobin concentration is 10.5 g/dL, leukocyte count is 15,600/μL, and urinalysis reveals 50–75 leukocytes/hpf and many bacteria/hpf. The patient has an increase anion gap metabolic acidosis. The patient is admitted to the intensive care unit and antibiotic therapy is started.

Which of the following in the next goal of therapy?

A)

Aggressive fluid resuscitation

 

B)

Hemodynamic monitoring with a pulmonary artery catheter

 

C)

Maintaining hemoglobin concentration above 12 g/dL (120 g/L)

 

D)

Maintaining PCO2 below 50 mmHg

 

E)

Vasopressor therapy

 

 




  • Answer: A


  • The patient has severe sepsis from pyelonephritis. Aggressive fluid resuscitation is indicated. Resolution of lactic acidosis within 6 h will have a beneficial effect on this patient’s survival. Resuscitation of the circulation should target a central venous oxygen saturation or mixed venous oxygen saturation of at least 70 %. Other goals include a central venous pressure of 8–12 mmHg, a mean arterial pressure of at least 65 mmHg, and a urine output of at least 0.5 mL/kg/h.


  • Fluid requirements are often as much as 5–6 L of fluid. Early goal-directed therapy sees the most benefits within the first 6 h. End points that improve survival include maintaining a SCVO2 of greater than 70 % and resolution of lactic acidosis.


  • Blood transfusion may be part of resuscitation for anemic patients in shock. In stable patients who have not had major blood loss or further blood loss is anticipated, a transfusion threshold of 7 g/dL is an acceptable approach. There are no data to support that maintaining a lower PCO2 is of any benefit. In addition placement of a pulmonary artery catheter would help to increase survival in this patient.




  • Reference


  • Rivers E, Nguyen B, Havstad S, Ressler J, et al. Early goal-directed therapy in the treatment of severe sepsis and septic shock. N Engl J Med. 2001;345(19):1368–77.


64.

A 52-year-old woman is admitted for a syncopal event while having her blood drawn. She has no past medical history and takes no medications. She experiences a brief loss of consciousness for about 20 s. She had no seizure-like activity and immediately returns to her usual level of functioning. You diagnose her with vasovagal syncope, and discharge the next day with no follow-up testing.

Which of the following statements regarding neurally mediated syncope is TRUE?

A)

Neurally mediated syncope occurs when there are abnormalities of the autonomic nervous system.

 

B)

Myoclonus does not occur during neurally mediated syncope.

 

C)

The final pathway of neurally mediated syncope results in a surge of the sympathetic nervous system with inhibition of the parasympathetic nervous system.

 

D)

The usual finding with cardiovascular monitoring is hypotension and tachycardia.

 

E)

The primary therapy for neurally mediated syncope is reassurance, avoidance of triggers, and plasma volume expansion.

 

 




  • Answer: E


  • Syncope accounts for 3 % of all emergency department visits and 1 % of all hospitalizations. Additionally, it is estimated that 35 % of all individuals will experience at least one syncopal event in their lifetime. Currently, no specific testing has sufficient power to be recommended for evaluation of syncope.


  • Syncope occurs when blood flow to the brain suddenly drops. Vasovagal syncope is one category without a clearly defined mechanism but can occur with intense emotions, strong odors, or orthostatic factors. Neurally mediated syncope can be brought about by specific mechanical events such as cough, micturition, swallowing, or carotid sensitivity. Reassurance and avoidance of triggers are the primary treatments. Liberal intake of fluids and salt and prevention of dehydration are protective against all forms of syncopal events.


  • In randomized controlled trials, isometric counterpressure maneuvers are also protective. In patients with refractory syncope, fludrocortisone, beta-blockers, and other vasoconstricting agents have been used with limited success. However, there are no clinical trial data to support their use.




  • References


  • Moya A, Sutton R, Ammirati F, et al. Guidelines for the diagnosis and management of syncope (version 2009): the Task Force for the Diagnosis and Management of Syncope of the European Society of Cardiology (ESC). Eur Heart J. 2009;30(21):2631–71.


  • Tan MP, Parry SW. Vasovagal syncope in the older patient. J Am Coll Cardiol. 2008;51(6):599–606.


65.

A 56-year-old woman who is listed for liver transplantation due to hepatitis C and portal hypertension is admitted for worsening ascites. She has had required one uncomplicated, large-volume paracentesis during the past 5 months. Her current medications are furosemide 40 mg daily and spironolactone 100 mg daily. She adheres to a daily diet containing less than 2000 mg of sodium and 2 L of free water.

On physical exam, her blood pressure is 115/78 mmHg. She has mild muscle wasting. A prominent second heart sound is noted on cardiac auscultation. The abdomen is protuberant with moderate ascites. No tenderness is noted. No lower extremity edema is evident, and the patient exhibits no confusion or asterixis.

Her sodium is 132 mmol/L, creatinine is 1.4 mg/dl.

Which of the following is the correct approach to treat this patients worsening ascites?

A)

Adjust fluid restriction to 1.5 L daily

 

B)

Antibiotics for possible bacterial peritonitis

 

C)

Continue serial paracenteses with albumin infusions

 

D)

Refer for transjugular intrahepatic portosystemic shunt

 

E)

Increase furosemide and spironolactone

 

 




  • Answer: E


  • It would be reasonable to try an increased diuretic dose in this patient. The recommended initial regimen is furosemide 40 mg plus spironolactone 100 mg daily. This patient is not at the maximum diuretic dose. Diuretics can be increased every 3–5 days, while maintaining the 40:100 mg ratio. In ascites due to end-stage liver disease, the maximum dose for furosemide is 160 mg daily and for spironolactone it is 400 mg daily. At the higher doses, decreasing efficacy will be seen. As the total diuretic dose is increased, it can be given once daily or divided as twice daily dosing.


  • Serum electrolytes and renal function tests should be carefully monitored as dose adjustments are made. The patient should discontinue diuretic therapy if the serum sodium decreases to less than 120 mmol/L, uncontrolled or recurrent encephalopathy develops, or the serum creatinine exceeds 2.0 mg/dL.




  • Reference


  • Runyon BA, AASLD Practice Guidelines Committee. Management of adult patients with ascites due to cirrhosis: an update. Hepatology. 2009;49:2087–107.


66.

A 76-year-old female is admitted for respiratory failure. She has a history of prior dementia. On her previous admissions, she has had episodes of delirium, which have resulted in prolonged hospitalization. On admission she is on no sedatives or antipsychotic meds. The family is concerned about the possibility of hospital-induced delirium and would like efforts be made to prevent this.

In an effort to reduce the incidence of delirium in this patient, which of the following should you undertake?

A)

Nighttime use of melatonin

 

B)

Mobilizing patient to the chair early

 

C)

Prophylactic use of rivastigmine

 

D)

Maintaining lights on for visual stimulation

 

 




  • Answer: B


  • Delirium is a common problem in the hospitalized patient, especially with a history of underlying dementia or psychiatric disease. So far, only simple interventions focused on maintaining normal environmental issues have been proven to be of any benefit. These include promoting sleep by decreasing nighttime stimuli, use of hearing aids and eyeglasses, and minimizing restraints. One study showed a reduction of sound during the night by using earplugs in patients in the ICU setting resulted in a decreased risk of delirium by 53 %.


  • Family can play a role in decreasing delirium. They can assist in orienting and reassuring the patient. Support from a familiar nurse and staff should also be encouraged.


  • There are no definitive studies that demonstrate the use of any prophylactic medicines, such as haloperidol or risperidone, provides any benefit. The use of these and other sedatives should be minimized. Pain can contribute to delirium as well as the excessive use of narcotics. Rivastigmine has been shown to worsen delirium in the hospitalized patient. Melatonin has had no clear clinical benefit in reduction of delirium. Early physical and occupational therapy may also be of some benefit.




  • Reference


  • Inouye SK, Bogardus ST, Charpentier PA, et al. A multicomponent intervention to prevent delirium in hospitalized older patients. N Engl J Med. 4 1999;340(9):669–76


67.

A 62-year-old male has been admitted for diabetic foot ulcer and associated cellulitis for the past 4 weeks. He has chronic diabetic kidney disease, hypertension, and type 1 diabetes mellitus. Medications are metformin, insulin, and lisinopril.

He is started on vancomycin and ciprofloxacin.

On physical examination, blood pressure is 145/90 mmHg. An area of erythema extends about 3 cm around a 3 × 3 cm ulcer on the right great toe. The area involved is tender warm and fluctuant.

Laboratory studies reveal an albumin of 2.9 g/dL, and a serum creatinine of 4.1 mg/dL. For acute renal failure, complement levels are checked and they are low. Urine studies show a urine sodium of 15 mEq/L. Urinalysis reveals 25 erythrocytes per HPF and 1–2 erythrocyte casts.

Which of the following is the most likely cause of this patient’s acute kidney injury?

A)

Diabetic nephropathy

 

B)

IgA nephropathy

 

C)

Post-infectious glomerulonephritis

 

D)

Membranous glomerulonephritis

 

E)

Drug-induced acute renal failure

 

 




  • Answer: C


  • This patient has post-infectious glomerulonephritis (PIGN). PIGN presents as an acute nephritic syndrome characterized by rapid onset of edema, hypertension, oliguria, and erythrocyte casts seen in the urine sediment. Low complement levels further suggest the exudative proliferative glomerulonephritis patterns can be seen by light microscopy on biopsy specimens.


  • Diabetic nephropathy does not explain the onset of this patient’s acute kidney injury. The decline of the glomerular filtration is predictable and usually no greater than 12–16 mL/min/1.73 m2 per year. Patients with IgA nephropathy may present with an episode of acute renal injury precipitated by infection. Gross hematuria is often seen. Adult patients with primary membranous glomerulonephritis frequently present with a nephritic picture. In these patients, the urine sediment can be active and reveal granular casts. Erythrocyte casts are not seen. In addition, complement levels are normal.


  • It has been suggested that PIGN was the cause of death of the composer Wolfgang Amadeus Mozart.




  • References


  • Rodriguez-Iturbe B, Musser JM. The current state of post streptococcal glomerulonephritis. J Am Soc Nephrol. 2008;19(10):1855–64.


  • Zegers RH, Weigl A. Steptoe A death of Wolfgang Amadeus Mozart: an epidemiologic perspective. Ann Intern Med. 2009;151(4):274–8, W96-7 (ISSN: 1539–3704).


68.

A 60-year-old male with end-stage liver disease was admitted for shortness of breath. He reports progressive ascites over the past few weeks and a low-grade temperature.

On physical exam, his lungs are clear upon auscultation. The abdomen is tender and distended. Oxygenation is 84 % by pulse oximetry, which improves slightly to 87 % with 5 l of oxygen. Chest X-ray reveals lung fields without infiltrate or pleural effusions. Computed tomography reveals no evidence of pulmonary embolism.

Paracentesis is performed which reveals a white blood cell count of 1,500 WBCs, of which 58 are neutrophils. The patient is started on antibiotics. Two days later, the patient continues with shortness of breath and marked hypoxia. Pulse oximetry is 93 % when supine but decreases to 84 % when sitting.

Which of the following studies is most likely to confirm patient’s cause of hypoxia?

A)

Ventilation perfusion scan of the lungs

 

B)

High contrast CT scan of the chest

 

C)

Pulmonary arteriography

 

D)

Lung biopsies

 

E)

Echocardiography with saline bubble contrast

 

 




  • Answer: E


  • This patient has hepatopulmonary syndrome. In these cases, contrast- or bubble-enhanced echocardiography will reveal a intrapulmonary shunt. Hepatopulmonary syndrome is associated with platypnea, which is increased dyspnea in the upright position, and orthodeoxia, which is increased hypoxia when transitioning from the lying to the standing position. Hypoxemia, in this case due to intrapulmonary shunt, is not significantly affected by an increase in inhaled O2 concentration. This syndrome may resolve with liver transplantation and does not exclude the patient from being considered for transplant.




  • Reference


  • Rodriguez-Roisin R, Krowka MJ. Hepatopulmonary syndrome – a liver-induced lung vascular disorder. N Engl J Med. 2008;358:2378–87.


69.

You have just admitted a 28-year-old man with a witnessed seizure. He has a prior history of seizure disorder. The event was witnessed by family members. His family describes movement of his right hand that spread to involve the entire arm. He did not lose consciousness.

On physical examination, sensation is intact in the affected limb, but his strength is 0 out of 5 in the musculature of the right hand. . His electrolytes and complete blood count are within normal limits. A toxicology screen is normal. A noncontrast CT scan of his head is unremarkable.

What is the best course of action at this time?

A)

Cerebral angiogram

 

B)

Magnetic resonance angiogram

 

C)

Reassess in a few hours

 

D)

Psychiatric evaluation

 

E)

Lumbar puncture

 

 




  • Answer: C


  • The patient has Todd’s paralysis, which may take minutes to many hours to return to normal. The abnormal motor movements that begin in a restricted area and then progress to involve a larger area are termed Jacksonian march. If his symptoms were to persist beyond several hours, it would be reasonable to investigate a different etiology of his hand weakness with imaging studies. The symptoms are too limited to suggest conversion disorder. Magnetic resonance angiogram or cerebral angiogram may be useful to evaluate for cerebrovascular disorders, if symptoms persist.




  • Reference


  • Gallmetzer P, Leutmezer F, Serles W, Assem-Hilger E, Spatt J, Baumgartner C. Postictal paresis in focal epilepsies–incidence, duration, and causes: a video-EEG monitoring study. Neurology. 2004;62(12):2160–4.


70.

A 22-year-old woman is admitted with fatigue of 1 week’s duration. She reports that she had a febrile illness 3 weeks ago, during which she experienced a transient rash and joint pain. She was treated for a possible urinary tract infection with ciprofloxacin. She works in a day care facility, where there has been an outbreak of a febrile illness with a rash during the past few weeks. The patient has a history of hereditary spherocytosis. On physical examination, she is pale, somewhat lethargic, but otherwise there are no significant findings.

Her laboratory tests show a hematocrit of 22 %, the reticulocyte count is 0.5 %.

Which of the following is the most likely diagnosis for this patient?

A)

Acute leukemia

 

B)

Glucose-6-phosphate dehydrogenase (G6PD) deficiency

 

C)

Systemic lupus erythematosus

 

D)

Hereditary spherocytosis in hemolytic crisis

 

E)

Aplastic crisis caused by parvovirus B19

 

 




  • Answer: E


  • This patient has a parvovirus B19 infection. This is the virus that causes the common childhood disease known as erythema infectiosum or fifth disease. It can cause aplastic crises in persons with hemolytic disorders, chronic anemia in immunocompromised hosts, and fetal loss in pregnant women.


  • The rash of erythema infectiosum usually appears without prodromal symptoms after an incubation period of 4–14 days. The rash starts as a fiery-red rash on both cheeks. It then extends as an erythematous maculopapular eruption on the proximal extremities and trunk in a reticular pattern. The rash may wax and wane for several weeks. Arthralgia and arthritis are seen in up to 80 % of infected adults.


  • Parvovirus B19 can cause an aplastic crises in patients who have sickle cell anemia, hereditary spherocytosis, thalassemia, and various other hemolytic anemias. These aplastic crises are abrupt in onset and are associated with giant pronormoblasts in the bone marrow. They generally resolve spontaneously after 1 or 2 weeks. In immunocompromised patients, acute infection may lead to viral persistence and chronic bone marrow suppression. In this patient, the anemia with a low reticulocyte count suggests a transient aplastic process and not a hemolytic crisis.




  • Reference


  • Servey JT, Reamy BV, Hodge J. Clinical presentations of parvovirus B19 infection. Am Fam Physician. 2007;75(3):373–6.


71.

A 55-year-old woman is admitted with epigastric abdominal pain. Initial clinical exam and laboratory findings are consistent with acute pancreatitis. She is admitted for aggressive hydration and observation. Lipase and amylase have decreased. Four days later she is doing better but pain and nausea still persist. A CT scan with contrast reveals necrotizing pancreatitis. The patient is hemodynamically stable, afebrile, with WBC of 11,000.

What should be done next?

A)

CT-guided aspiration for culture and gram stain

 

B)

Repeat CT scan in 48 h

 

C)

Immediate initiation of broad-spectrum antibiotics

 

D)

Referral to general surgery for immediate debridement

 

E)

Continued observation and hydration

 

 




  • Answer: E


  • The current recommendations do not support the use of prophylactic antibiotics to prevent pancreatic infection among patients with necrotizing pancreatitis. Some studies suggest that the use of potent antibiotics may lead to a superimposed fungal infection. The current guidelines recommend medical management during the first 2–3 weeks. After 3 weeks, if symptoms persist or clinical condition deteriorates, a surgical debridement should be considered. If symptoms worsen or fail to improve, repeat imaging or biopsy can be considered.




  • References


  • Telem DA, Bowman K, Hwang J, Chin EH, Nguyen SQ, Divino CM. Selective management of patients with acute biliary pancreatitis. J Gastrointest Surg. 2009;13(12):2183–8.


  • Tenner S, Baillie J, Dewitt J, et al. American College of Gastroenterology guidelines: management of acute pancreatitis. Am J Gastroenterol. 2013;108(9):1400–15.


72.

You are asked to admit a 49-year-old female with the acute onset of fever and severe headache. Her past medical history is significant for renal transplant due to diabetes mellitus type 1. While in the emergency room, she develops chills, photophobia, and stiffness of her neck.

On physical exam, temperature is 38.6 °C (101.2 °F), heart rate is 90, and blood pressure is 120/68 mmHg

You have significant concern for meningitis.

Which of the following is NOT an appropriate next step in the patient’s management?

A)

Draw stat blood cultures while placing the orders for empiric antibiotics.

 

B)

Perform stat lumbar puncture while waiting for MRI.

 

C)

Check CT scan of brain without contrast before lumbar puncture.

 

D)

Give dexamethasone with first dose of antibiotics.

 

E)

A than D

 

 




  • Answer: E


  • It is important to recognize the treatment sequence in the management of suspected bacterial meningitis. Imaging and lumbar puncture should not delay empiric antibiotic coverage and steroids. The necessity of a CT scan prior to a lumbar puncture in all instances has been debated. The management algorithm for adults with suspected bacterial meningitis per Infectious Disease Society of America (IDSA) guidelines is as follows:

    1.

    Blood cultures STAT.

     

    2.

    Begin dexamethasone + empiric antimicrobial therapy.

     

    3.

    Check CT scan of the head before performing lumbar puncture.

     




  • Reference


  • Tunkel AR, Hartman BJ, Kaplan SL, Kaufman BA, Roos KL, Scheld WM, Whitley RJ. Practice Guidelines for the management of bacterial meningitis. IDSA. Clin Infect Dis. 2004;39:1267–84.


73.

A 68-year-old female with metastatic breast cancer involving the lungs is admitted for increasing agitation. She has been enrolled in home hospice for the past 4 weeks. She has diffuse back pain, which is moderately well controlled with transdermal fentanyl and oral hydromorphone.

On physical exam, the patient is frail and cachectic. Tachycardia is noted. She is neurologically intact as well as alert and oriented. Her daughter tells you that she produces a small amount of concentrated urine a few times daily and that she eats occasional small meals but is often nauseated. Her daughter notes that she becomes agitated just before dawn if she is still awake. The daughter would like to take the patient home if the behavior can be controlled.

Which of the following should you do to decrease this patient’s agitation?

A)

Prescribe lorazepam on an as-needed basis.

 

B)

Prescribe zolpidem at bedtime.

 

C)

Prescribe an evening dose of quetiapine.

 

D)

Request that the hospice social worker meets with the patient to address her fears and worries.

 

E)

Admit to inpatient hospice.

 

 




  • Answer: C


  • Quetiapine can be used for agitation in critically ill patients. In a prospective, randomized, double-blind, placebo-controlled study conducted on 36 adult critically ill patients with delirium, quetiapine in escalating doses was shown to be effective in palliating agitation.


  • The use of lorazepam may worsen her agitation in her medically fragile state. The use of zolpidem will address this patient’s disrupted sleep but may increase her agitation. Supporting the family with rapid control of behavior issues is needed. While inclusion of psychosocial support is helpful, this patient is rapidly declining and medication can provide immediate benefit.




  • References


  • Devlin JW, Roberts RJ, Fong JJ, et al. Efficacy and safety of quetiapine in critically ill patients with delirium: a prospective, multicenter, randomized, double-blind, placebo-controlled pilot study. Crit Care Med. 2010;38(2):419–27.


  • Larson AM, Polson J, Fontana RJ, Davern TJ, Lalani E, Hynan LS, Reisch JS, Schiødt FV, Ostapowicz G, Shakil AO, Lee WM, Acute Liver Failure Study Group. Acetaminophen-induced acute liver failure: results of a United States multicenter, prospective study. Hepatology. 2005;42(6):1364–72.


74.

A 34-year-old female is admitted with a 2-day history of right eye redness and pain, photophobia, and decreased visual acuity. She has a 2-year history of recurrent oral ulcerations and tender nodules on her shins. She has had mild rotating joint tenderness for the past 4 months. Her only medication is ibuprofen. She has been treated for genital herpes in the past.

On physical examination, the temperature is 37.4 °C (99.3 °F), blood pressure is 130/80 mmHg, pulse rate is 110/min, and respiration rate is 16/min. Oral ulcerations are noted on the inner cheek, palate, and tongue. The lungs are clear. The abdomen is nontender. No bruits are noted. The right knee and right ankle are swollen. Peripheral pulses are normal. An ophthalmology consultation reveals anterior and posterior uveitis.

Complete blood count and the basic metabolic panel and INR are within normal limits. Chest X-ray reveals a prominent right pulmonary artery. CT of the chest demonstrates an aneurysm of the right pulmonary artery.

Which of the following is the most likely diagnosis?

A)

Behçet disease

 

B)

Granulomatosis with polyangiitis

 

C)

Polyarteritis nodosa

 

D)

Gonococcal arthritis

 

E)

Reiter’s syndrome

 

 




  • Answer: A


  • This patient has Behçet disease. Behçet disease is characterized by the triad of recurrent oral aphthous ulcers, genital ulcers, and uveitis.


  • Behçet disease is a systemic disorder characterized by vasculitis and multiple organ involvement. The diagnostic clues are intermittent mucous membrane ulcerations and ocular involvement. Gastrointestinal, pulmonary, musculoskeletal, and neurologic manifestations may be present. This patient has a 2-year history of recurrent oral ulcerations. The skin lesions are erythema nodosum. She now presents with panuveitis. Pulmonary artery aneurysm also strongly suggests Behçet disease.


  • Exposure to an infectious agent may trigger a cross-reactive immune response. Proposed infectious agents have included herpes simplex virus, Streptococcus species, Staphylococcus species, and Escherichia coli. There may be relationship to flora of the mouth.


  • The treatment approach depends on the severity of the disease and major organ involvement. This may include systemic corticosteroids, azathioprine, pentoxifylline, dapsone, interferon-alfa, colchicine, and thalidomide.




  • References


  • Hatemi G, Silman A, Bang D, et al. EULAR recommendations for the management of Behçet disease. Ann Rheum Dis. 2008;67(12):1656–62.


  • Study Group for Behçet’s Disease. Criteria for diagnosis of Behçet’s disease. International Study Group for Behçet’s Disease. Lancet. 1990;335(8697):1078–80.


75.

A 58-year-old female is admitted for nausea, vomiting, and a diffuse rash. Four days before admission, she was bitten on her hand by her neighbor’s dog. The patient reports no other symptoms. Ten years ago, she underwent splenectomy following a motor vehicle collision. On physical exam, temperature is 36.8 °C (98.2 °F), pulse rate is 90 per minute, respirations are 16 per minute, and blood pressure is 120/76 mmHg. The patient appears alert and cooperative. The neck is supple. The lungs are clear. Two deep lesions are noted on the dorsum of the left hand. Faint purple, macular lesions are seen on the trunk and extremities. The lesions are not compressible, painful, or pruritic.

The leukocyte count is 17,000/μL with 15 % band forms . Despite rapid administration of intravenous fluids, vancomycin, piperacillin/tazobactam, and clindamycin, the patient’s blood pressure drops and is transferred to the ICU for vasopressor support.

Which of the following organisms is the most likely cause of the findings in this patient?

A)

Pasteurella multocida

 

B)

Neisseria meningitidis

 

C)

Streptococcus pneumoniae

 

D)

Capnocytophaga canimorsus

 

 




  • Answer: D


  • This patient has disseminated C. canimorsus infection due to a dog bite and asplenia. C. canimorsus is a normal colonizing bacterium of dog and cat saliva. Canimorsus is Latin for “dog bite.”


  • Clinical symptoms usually begin 5–6 days after the dog bite, scratch, or other exposure. Patients typically present with signs of sepsis including fever, vomiting, and abdominal pain. There is progressive cutaneous hemorrhage with infarction that leads to extensive skin necrosis.


  • Several other organisms can produce purpura fulminans including endotoxin-producing Neisseria meningitidis and encapsulated Streptococcus pneumoniae and Haemophilus influenzae.




  • References


  • Pers C, Gahrn-Hansen B, Frederiksen W. Capnocytophaga canimorsus septicemia in Denmark, 1982–1995: review of 39 cases. Clin Infect Dis. 1996;23(1):71–5.


  • Eefting M, Paardenkooper T. Capnocytophaga canimorsus sepsis. Blood. 2010;116(9):1396.


76.

A 28-year-old male is admitted with acute agitation. He was brought to the hospital by emergency medical services, which was called by his neighbor. He was found on the roof of his house. Little is known of his past medical history.

On physical exam, he is actively hallucinating, diaphoretic, and is nonresponsive to painful stimuli. In addition to emergency room staff, he requires four security guards to restrain. Drug screen is negative. Computed tomography of his head is not possible due to agitation. Over the course of next 2 days, he requires large doses of benzodiazepines and haloperidol for management and sedation. In addition he requires physical restraints. He gradually returns to his usual functional status in 5 days with little recollection of the event.

The most likely ingested substance was:

A)

Synthetic cathinones “bath salts”

 

B)

Hallucinogenic mushrooms

 

C)

Heroin

 

D)

Cocaine

 

E)

Ecstasy

 

 




  • Answer: A


  • Synthetic cathinones, drugs known as “bath salts,” were first described in the United States in 2010. Users of bath salts experience vivid disturbing hallucinations, agitation, paranoia, and extreme pain intolerance. They are more potent as compared to other hallucinogens. Episodes of intoxication are unpredictable and are often prolonged lasting several days. Large dose of sedatives are often required as well as careful use of physical restraints. Decreased pain sensation makes physical restraint difficult and reports of injury during sedation are common.




  • Reference


  • Coppola M, Mondola R. Synthetic cathinones: Chemistry, pharmacology and toxicology of a new class of designer drugs of abuse marketed as “bath salts” or “plant food”. Toxicol Lett. 2012;211(2):144–9. doi:10.​1016/​j.​toxlet.​2012.​03.​009. PMID 22459606.


77.

A 22-year-old male college student is admitted for having a witnessed generalized tonic-clonic seizure. This occurred in the morning as witnessed by his roommate.

The patient reports that he was out late the night before and drank more than usual over the course of the evening. He reports having sudden jerks of his arms this morning before the generalized seizure was witnessed. He has had similar muscular jerks in the previous mornings. This has particularly occurred on days when he has little sleep. He reports no history of excessive alcohol use or illicit substance abuse. He takes no medications.

Neurologic examination is normal. He is oriented and feeling well the day after admission.

Results of laboratory studies are normal. A CT scan of the head shows no abnormalities.

Which of the following is the most likely diagnosis?

A)

Alcohol withdrawal seizure

 

B)

Benign rolandic epilepsy

 

C)

Illicit drug-induced seizure

 

D)

Temporal lobe epilepsy

 

E)

Juvenile myoclonic epilepsy

 

 




  • Answer: E


  • This patient has juvenile myoclonic epilepsy. A history of rapid, unprovoked jerks and generalized tonic-clonic seizures on awakening is a common presentation. Onset is usually in adolescence, but may occur in an early adulthood.


  • Juvenile myoclonic epilepsy may affect 5–10 % of all patients with epilepsy. Seizures are often provoked by sleep deprivation, alcohol, video games, or exposure to flickering lights.


  • Recognizing the specific epilepsy syndrome affecting a patient is important in selecting the appropriate therapy.


  • Alcohol withdrawal seizures develop in chronic users of alcohol. It is generally seen in combination with other signs and symptoms of alcohol withdrawal, such as delirium, tremor, tachycardia, and diaphoresis.


  • Benign rolandic epilepsy is a syndrome seen in younger children who have seizures, usually during sleep. Temporal lobe epilepsy is the most common of the localization-related epilepsies. This often is due to a specific brain malformation, such as trauma, infarct, or congenital. The most common seizure occurring with temporal lobe epilepsy is complex partial seizure. Patients with complex partial seizures are awake but exhibit altered awareness, such as unresponsiveness or staring.




  • References


  • Prasad A, Kuzniecky RI, Knowlton RC, et al. Evolving antiepileptic drug treatment in juvenile myoclonic epilepsy. Arch Neurol. 2003;60(8):1100–5


  • Proposal for revised classification of epilepsies and epileptic syndromes. Commission on Classification and Terminology of the International League Against Epilepsy. Epilepsia. 1989;30(4):389–99.


78.

A 38-year-old female is admitted to the hospital for a 2-day history of fever and abdominal pain. Her medical history is notable for cirrhosis due to chronic hepatitis C, esophageal varices, and ascites. Her medications are furosemide, spironolactone, nadolol, and lactulose.

On physical examination, the temperature is 36.5 °C (97.7 °F), blood pressure is 110/60 mmHg, pulse rate is 90/min, and respiration rate is 20/min. Abdominal examination discloses distention. The abdomen is mildly tender upon palpation.

Laboratory studies show hemoglobin of 9 g/dL, leukocyte count 3,700/μL platelet count 82,000/μL, INR 1.6, albumin 2.3 g/dL, alkaline phosphatase 162 units/L, alanine aminotransferase 27 units/L, aspartate aminotransferase 32 units/L, total bilirubin 3.8 mg/dL, and creatinine 2.4 mg/dL. Abdominal ultrasound reveals cirrhosis, splenomegaly, and ascites. Diagnostic paracentesis discloses a cell count of 1,700/μL with 20 % neutrophils, a total protein level of 1.2 g/dL, and an albumin level of 0.7 g/dL.

Which of the following is the most appropriate treatment?

A)

Cefotaxime

 

B)

Cefotaxime and albumin

 

C)

Furosemide and spironolactone

 

D)

Large-volume paracentesis

 

E)

Vancomycin and cefotaxime

 

F)

Ciprofloxacin

 

 




  • Answer: B


  • This patient has spontaneous bacterial peritonitis and acute kidney injury . The diagnosis of spontaneous bacterial peritonitis (SBP) is made in the setting of an elevated ascitic fluid absolute polymorphonuclear (PMN) cell count of greater than 250/μl without evidence of secondary causes of peritonitis. A positive bacterial culture of the ascitic fluid is not needed. Intravenous cefotaxime or a similar third-generation cephalosporin is the treatment of choice for SBP. However per Cochrane review, this class has not been shown to be superior to other classes of antibiotics. Most common isolates are Escherichia coli, Klebsiella pneumoniae, and pneumococci. Vancomycin is not needed for initial treatment. Oral fluoroquinolone treatment may be indicated in mild cases treated as an outpatient.


  • Several strategies may be employed to improve renal vascular flow in the setting of SBP. Intravenous albumin is the most widely used.


  • The use of cefotaxime plus intravenous albumin at 1.5 g/kg on day 3 has been shown to decrease in-hospital mortality by 20 % in patients with serum creatinine values of 1.5 mg/dL or greater. There is no evidence that large-volume paracentesis improves outcomes in patients with SBP and should be done with caution. Excessive fluid shifts may worsen kidney function.




  • References


  • Chavez-Tapia NC, Soares-Weiser K, Brezis M, Leibovici L. Antibiotics for spontaneous bacterial peritonitis in cirrhotic patients. Cochrane Database Syst Rev. 2009;1:CD002232.


  • Sort P, Navasa M, Arroyo V, et al. Effect of intravenous albumin on renal impairment and mortality in patients with cirrhosis and spontaneous bacterial peritonitis. N Engl J Med. 1999;341(6):403–9.


79.

A 55-year-old female presents to the emergency department 5 h after the onset of left hemiplegia and right gaze deviation. CT scan reveals an early large infarct. Her airway appears to be intact and she is arousable. She responds to voice commands appropriately. She is admitted to the hospitalist service with a neurology consult.

Ten hours later, the patient becomes somnolent. On repeat examination, she is no longer responsive to voice and has minimal withdrawal to pain. The right pupil is large, irregular, and unresponsive. Repeat CT scan of the head reveals a 10 mm midline shift as well as the evolution of a well-demarcated right middle cerebral artery infarction.

Which of the following is the most appropriate next step in her treatment and management?

A)

Neurosurgical consultation for possible hemicraniectomy

 

B)

Dexamethasone intravenously

 

C)

Transfer to the intensive care unit for intracranial pressure monitoring

 

D)

Aspirin

 

E)

Bedside intubation

 

 




  • Answer: A


  • Patients who have a large territory infarcts are at risk for herniation and should have frequent neurologic checks to follow for signs of deterioration. Early repeat imaging and neurosurgical consult is warranted with significant clinical decline. Three separate European studies reveal that hemicraniectomy reduces mortality and severe disability in patients with malignant middle cerebral artery infarction. This benefit is greatest if performed within the first 48 h after stroke and optimally before clinical herniation has occurred. This patient has evidence of elevated clinical intracranial pressure, and urgent neurosurgical consultation is needed as well as repeat imaging. The neurosurgical consult should come first while arranging for imaging, intensive care unit transfer, and further supportive measures.




  • Reference


  • Gupta R, Connolly ES, Mayer S, Elkind MSV. Hemicraniectomy for massive middle cerebral artery territory infarction: a systematic review. Stroke. 2004;35:539–43.


80.

A 54-year-old male is admitted for observation after suffering a concussion in a syncopal episode and suffering a laceration of the head. This occurred while getting up at night to go to the bathroom. On presentation he continues to feel slightly dizzy and is noted to be dehydrated.

He has no past medical history and no meds. He also reports increased thirst and urination for the past month. Urinalysis reveals 2+ glucose. His blood sugar is 305 mg/dl. His hemoglobin A1C is 11 %. He is started on intravenous fluids in the emergency room.

What is an appropriate initial diabetic regimen for this patient?

A)

Metformin 500 mg PO BID

 

B)

Insulin

 

C)

Januvia 100 mg PO daily

 

D)

Metformin 500 mg PO BID and glipizide 5 mg PO daily

 

 




  • Answer: B


  • Initial diabetic therapy is guided by hemoglobin A1C and symptoms. According to the American Diabetes Association (ADA), the recommended goal A1C for this patient is less than 7 %. Metformin as monotherapy, if not contraindicated, may be first-line therapy. However, in newly diagnosed type 2 diabetics with marked symptoms and/or highly elevated blood glucose or A1C, insulin therapy is indicated.


  • The American Association of Clinical Endocrinology has more specific guidelines with respect to initiating therapy. Specifically, when the entry level A1C is greater than 9 % and/or the patient is symptomatic (urinalysis 2+ glucose, polydipsia, polyuria), insulin plus or minus other agents is recommended. Monotherapy is recommended when the entry A1C is less than 7.5 %. Similarly, dual therapy is recommended when the entry level A1C is greater than or equal to 7.5 %, but less than 9 %. Additionally, the oral agents will drop the A1C by approximately 1–2 % (not all oral agents); therefore, this would not be an adequate decrease for a patient with an A1C of 9 % or greater.




  • References


  • ADA Professional Practice Committee. Clinical practice recommendations. Diabetes Care 2014;37: S1–155.


  • American Association of Clinical Endocrinology. Comprehensive diabetes management algorithm 2013. Endocr Pract. 2013;19:1–48.


81.

A 50-year-old male is admitted due to observation for chest pain and the possibility of myocardial ischemia. On presentation, he reported a brief episode of burning chest pain that occurred with maximal exertion. The pain lasted less than a minute.

Since admission, he has had no further chest pain. He has a positive family history for coronary artery disease. He is currently a 1-pack/day smoker. On the first day of admission, blood pressure is noted to be 180/100.

Which of the following is the most appropriate medicine for the treatment of his hypertension?

A)

Nifedipine

 

B)

Labetalol

 

C)

Clonidine

 

D)

Enalapril

 

E)

Hydrochlorothiazide

 

 




  • Answer: B


  • Beta-blockers are the best initial antihypertensive agent to use when the possibility of cardiac ischemia is present. This would be a reasonable first-line choice until the possibility of coronary artery disease is explored.




  • Reference


  • Marik PE, Varon J. Hypertensive crises: challenges and management. Chest. 2007;131(6):1949–62.


82.

A-65-year-old male with a past medical history of hypertension is admitted with the diagnosis of a pulmonary embolism and is started on intravenous heparin.

After being transported to his room, his heart rate increases to 130 bmp with a blood pressure of 100/60 mmHg. An ECG reveals atrial fibrillation with rapid ventricular response. Rate control is attempted with three doses of IV 5 mg Lopressor, but failed to decrease heart rate. A Cardizem drip is started.

While at the bedside the patient’s heart rate increases to 150 bpm and he begins to complain of chest pain. A repeat blood pressure is checked and noted to be 80/40. You begin a 500 cc bolus of normal saline but the patient experiences a syncopal episode while sitting in bed. Pulses are faintly palpable and respirations remain intact. A cardiac code is called.

What is the most appropriate next step in acute management after activating the emergency response team?

A)

Place the patient in reverse Trendelenburg.

 

B)

Begin chest compressions at a rate of 30:2.

 

C)

Push Lopressor 5 mg IV × 1 dose.

 

D)

Immediate R-wave synchronized direct-current cardioversion.

 

 




  • Answer: D


  • When a rapid ventricular response does not respond promptly to pharmacologic measures for patients with atrial fibrillation with ongoing myocardial ischemia, symptomatic hypotension, angina, or heart failure, immediate R-wave synchronized direct-current cardioversion is recommended.




  • Reference


  • Anderson J et al. Management of patients with atrial fibrillation (compilation of 2006 ACCF/AHA/ESC and 2011 ACCF/AHA/HRS recommendations): a report of the American College of Cardiology/American Heart Association Task Force on Practice Guidelines. Circulation. 2013;127:1916–26.


83.

A 75-year-old man was admitted to the hospital for diarrhea and hypotension. During the past year, he has had four prior admissions because of similar problems. Polymerase chain reaction assay comes back positive for Clostridium difficile infection. This is his third confirmed Clostridium difficile infection documented by stool polymerase chain reaction assay.

On physical exam, the patient’s blood pressure is 80/40 mmHg. The abdomen has diffuse tenderness without peritoneal signs. Computed tomography scan of the abdomen did not show any bowel dilation. He is started on oral vancomycin and IV Flagyl.

Since admission, his hemodynamic profile has rapidly stabilized with additional fluid administration.

Which of the following should be considered now?

A)

Rifaximin

 

B)

Neomycin enema

 

C)

Assessment for presence of vancomycin-resistant enterococci

 

D)

Fecal microbiota transplantation

 

E)

Probiotics

 

 




  • Answer: D


  • Recurrent Clostridium difficile infection can be life threatening. In this patient, the diagnosis is established, and colonoscopy is unlikely to yield additional results of value. Fecal microbiota transplantation should be considered. Initial experience with a fecal transplant is promising, and certainly in a case of multiple recurrences such as this, it should be considered.


  • Both initial and sustained responses to fecal microbiota transplant for the treatment of refractory C. difficile infection remain high out to 18 months follow-up.




  • References


  • Bakken JS. Fecal bacteriotherapy for recurrent Clostridium difficile infection. Anaerobe 2009; 15:285–289.


  • Crooks NH, Snaith C, Webster D, et al. Clinical review: probiotics in critical care. Crit Care. 2012;16:237.


84.

A 65-year-old male with end-stage liver disease secondary to alcohol presents with a chief complaint of worsening ascites.

On physical examination, minimal ascites is noted. Blood pressure is 90/50. Chemistries reveal a creatinine of 5.3 mg/dL and a BUN of 42 mg/d/L. Urine sodium is noted to be 5 mEq/L. Urine volume in the first 24 h of admission is 120 mL/day. No red blood cells are noted on initial exam.

Initial treatment includes which of the following?
Mar 26, 2017 | Posted by in GENERAL & FAMILY MEDICINE | Comments Off on Inpatient Medicine

Full access? Get Clinical Tree

Get Clinical Tree app for offline access